ASIPP Anatomy Questions Flashcards

1
Q
1. Sacral splanchnics are primarily:
A. Sympathetic postganglionic fi bers
B. Parasympathetic postgangalionic fi bers
C. Sympathetic preganglionic fi bers
D. Parasympathetic preganglionic fi bers
E. Senory preganglionic fi bers
A
  1. Answer: A
    Explanation:
    Sacral splanchnics refer to sympathetic fi bers traveling to
    the hypogastric plexuses. Another name for the superior
    hypogastric plexus is the presacral nerve.
    Source: Boswell MV, Board Review 2005
How well did you know this?
1
Not at all
2
3
4
5
Perfectly
2
Q
2. The most common form of inguinal hernia begins
A. At the deep inguinal ring
B. Medial to the epigastric artery
C. Travels through the inguinal triangle
D. Entraps the genitofemoral nerve
E. Rarely enters the scrotum
A
  1. Answer: A
    Explanation:
    Direct inguinal hernias pass through the inguinal triangle,
    which is an area of weak fascia. They almost never go into
    the scrotum. Both the superfi cial inguinal ring and the
    inherent weakness of abdominal wall lateral to the falx
    inguinalis make this area susceptible to hernias. Indirect
    inguinal hernias start at deep inguinal ring, pass down
    inguinal canal, through superfi cial ring, and, in the male,
    usually descend into scrotum along with the spermatic
    cord. In the female they travel along the round ligament.
    Source: Boswell MV, Board Review 2005
How well did you know this?
1
Not at all
2
3
4
5
Perfectly
3
Q
3. The obturator nerve innervates which of the following
muscles?
A. Gracilis
B. Sartorius
C. Rectus Femoris
D. Pectineus
E. Vastus Medialis
A
  1. Answer: A

Source: Day MR, Board Review 2005

How well did you know this?
1
Not at all
2
3
4
5
Perfectly
4
Q
  1. What is true about the superfi cial cervical plexus?
    A. Blocks of this plexus are useful for upper airway laryngoscopy
    B. One branch is the suprascapular nerve
    C. One branch is the lesser occipital nerve
    D. Needle insertion is between the anterior and middle
    scalene muscles
    E. There is a very high risk for intrathecal injections
A
  1. Answer: C
    Explanation:
    (Raj, Pain Medicine Review, 2nd Ed., pages 232-236)
    Blocks of the superior laryngeal nerve are useful for upper
    airway laryngoscopy (above the vocal cords). Blocks of the
    recurrent laryngeal nerve are useful for tracheal
    procedures (below the vocal cords). The suprascapular
    nerve originates from C5,6 and goes posteriorly to
    innervate the supraspinatus and infraspinatus (with
    sensory input from the shoulder joint). The lesser
    occipital, greater auricular, transverse cervical, and
    supraclavicular nerves originate from C2, C3, C4 and
    comprise the superfi cial cervical plexus. An interscalene
    brachial plexus block is performed through the anterior
    and middle scalene muscles. There is a low risk of
    intrathecal injections compared to a deep cervical plexus
    block.
    Source: Shah RV, Board Review 2005
How well did you know this?
1
Not at all
2
3
4
5
Perfectly
5
Q
  1. Which of the following is true about the auriculotemporal
    nerve?
    A. It is a branch of the mandibular division of trigeminal
    ganglion
    B. It is anterior to the superfi cial temporal artery
    C. It is posterior to the external auditory meatus
    D. Blocking it would alleviate pain when the earlobe undergoes
    piercing
    E. It provides sensation to the cheek
A
  1. Answer: A
    Explanation:
    (Raj, Pain Review 2nd Ed., page 229)
    The auriculotemporal nerve originates from the V3 branch
    of the trigeminal ganglion. It travels posterior to the
    superfi cial temporal artery and anterior to the external auditory meatus. It receives sensation from the TMJ,
    parotid gland, external auditory meatus, tympanic
    membrane, tragus (not earlobe), and skin over the
    temporal area.
    Source: Shah RV, Board Review 2005
How well did you know this?
1
Not at all
2
3
4
5
Perfectly
6
Q
  1. You perform a glossopharyngeal nerve block. Which of the
    following is not likely to be related to the block?
    A. Torticollis
    B. Seizure
    C. Hoarseness
    D. Dysphagia
    E. Diffi culty with smiling
A
  1. Answer: E
    Explanation:
    (Raj, Pain Medicine Review, 2nd Ed. Page 232)
    The glossopharyngeal nerve exits the jugular foramen
    along with cranial nerves X and XI. Hence one can develop
    dysphagia (IX), hoarseness (X), and torticollis (XI).
    Seizures can occur as a consequence of intra-arterial
    injection into the carotid. Diffi culty with smiling occurs
    with palsy of cranial nerve VII which exits through the
    stylo-mastoid foramen.
    Source: Shah RV, Board Review 2005
How well did you know this?
1
Not at all
2
3
4
5
Perfectly
7
Q
7. Blockade of the brachial plexus via the interscalene
approach commonly misses what nerve?
A. Median
B. Axillary
C. Radial
D. Ulnar
E. Musculocutaneous
A
  1. Answer: D

Source: Day MR, Board Review 2005

How well did you know this?
1
Not at all
2
3
4
5
Perfectly
8
Q
8. Lower esophageal pain can be improved by blocking spinal
nerve roots at which levels?
A. T2 - T3
B. T3 - T5
C. T5 -T9
D. T10 - T11
E. T11 - T12
A
  1. Answer: C
    Explanation:
    Innervation by lower thoracic splanchnics and vagus
    afferents
    Raj. Chapter 43. Thoracoabdominal Pain. In: Practical
    Management of Pain 3rd Edition, Raj et al, Mosby, 2000
    Source: Boswell MV, Board Review 2005
How well did you know this?
1
Not at all
2
3
4
5
Perfectly
9
Q
9. Of the following nerve block techniques, which one
produces anesthesia of ALL of terminal branches of the
brachial plexus?
A. Interscalene
B. Supraclavicular
C. Axillary
D. Deep cervical
E. Superfi cial cervical
A
  1. Answer: B

Source: Day MR, Board Review 2005

How well did you know this?
1
Not at all
2
3
4
5
Perfectly
10
Q
10. Structures innervated by the inferior hypogastric plexus
include
A. Uterus
B. Bladder
C. Rectum
D. Scrotum
E. Vagina
A
  1. Answer: A
    Explanation:
    The uterus is innervated by the sympathetics passing
    inferiorly via the inferior hypogastric plexus.The other
    organs listed are innervated by the pelvic splanchnics,
    arising from the sacral roots or the pudendal nerve.
    Source: Boswell MV, Board Review 2005
How well did you know this?
1
Not at all
2
3
4
5
Perfectly
11
Q
11. Which of the following is the most important center of
sympathetic and parasympathetic distribution to the
pelvis?
A. Superior hypogastric plexus
B. Inferior hypogastric plexus
C. Pelvic splanchnic nerves
D. Pudendal nerve
E. Nervi erigentes
A
  1. Answer: B
    Explanation:
    The superior hypogastric plexus sends sympathetics to the
    inferior hypogastric plexus, the inferior hypogastric
    plexus receives parasympathetics from the pelvic
    sphlancnics
    Source: Boswell MV, Board Review 2005
How well did you know this?
1
Not at all
2
3
4
5
Perfectly
12
Q
12. The web space between the great toe and the second toe is
innervated by what nerve?
A. Sural
B. Saphenous
C. Superfi cial peroneal
D. Deep peroneal
E. Posterior tibial
A
  1. Answer: D
    Explanation:
    The deep peroneal nerve innervates the short extensors of
    the toes and the skin of the web space between the great
    and second toe.
    The deep peroneal nerve is blocked at the ankle by
    infi ltration between the tendons of the anterior tibial and
    extensor hallucis longus muscle.
    Source: Day MR, Board Review 2005
How well did you know this?
1
Not at all
2
3
4
5
Perfectly
13
Q
  1. The preganglion cell bodies of the sympathetic nervous
    system are located where in the spinal cord?
    A. Dorsal columns
    B. Lateral spinothalamic tract
    C. Intermediolateral cell column
    D. Substantia gelatinosa
    E. Rexed laminae IV
A
  1. Answer: C

Source: Day MR, Board Review 2005

How well did you know this?
1
Not at all
2
3
4
5
Perfectly
14
Q
14. Stellate ganglion is:
A. Combination of C7-T1 ganglion
B. Combination of C8-T1 ganglion
C. Combination of C7-C8
D. Sits on C6 transverse process
E. Located under the longus coli muscle
A
  1. Answer: A

Source: Racz G. Board Review 2003

How well did you know this?
1
Not at all
2
3
4
5
Perfectly
15
Q
  1. Anatomically, the celiac plexus is located anterolateral to
    the aorta at which level?
    A. Above the diaphragm at T10
    B. At the crura of the diaphragm at L1
    C. At the bifurcation of the aorta at L4
    D. Above the crura of the diaphragm at T12
    E. Surrounding then superior mesenteric artery
A
  1. Answer: B

Source: Boswell MV, Board Review 2005

How well did you know this?
1
Not at all
2
3
4
5
Perfectly
16
Q
  1. The lateral cord of the brachial plexus is formed by which
    of the following divisions?
    A. Anterior divisions of the superior and middle trunks
    B. Posterior divisions of the superior, middle, and inferior
    trunks
    C. Anterior division of the inferior trunk
    D. Anterior division of the superior trunk and posterior
    division of the middle trunk
    E. Posterior divisions of the middle and inferior trunks
A
  1. Answer: A

Source: Day MR, Board Review 2005

How well did you know this?
1
Not at all
2
3
4
5
Perfectly
17
Q
17. T5-T12 sympathetic ganglia transmit nociceptive afferents
from the upper abdominal organs with which of the
following?
A. Renal plexus
B. Celiac plexus
C. Cardiac plexus
D. Pulmonary plexus
E. Hypogastric plexus
A
  1. Answer: B

Source: Boswell MV, Board Review 2005

How well did you know this?
1
Not at all
2
3
4
5
Perfectly
18
Q
18. In terms of surface anatomy, a line connecting the inferior poles of both scapulae would intersect which vertebral
body?
A. C7
B. T3
C. T7
D. T10
E. T12
A
18. Answer: C
Explanation:
(Raj, Pain Review 2nd Ed., page 216)
The line connecting the inferior poles of both scapula
would intersect T7.
Source: Shah RV, Board Review 2005
How well did you know this?
1
Not at all
2
3
4
5
Perfectly
19
Q
  1. The purpose of the cavernous nerves of the penis is to
    provide:
    A. Sympathetics and allow for erection
    B. Parasympathetics and allow for ejaculation
    C. Parasympathetics and allow for erection
    D. Sympathetics and allow for ejaculation
    E. Sensory afferent muscular tone
A
  1. Answer: C
    Explanation:
    Parasympathetic is point and sympathetic is shoot.
    Source: Boswell MV, Board Review 2005
How well did you know this?
1
Not at all
2
3
4
5
Perfectly
20
Q
  1. Which of the techniques for blockade of the brachial
    plexus has the highest incidence of pneumothorax?
    A. Interscalene
    B. Supraclavicular
    C. Infraclavicular
    D. Axillary
    E. Deep Cervical
A
  1. Answer: B

Source: Day MR, Board Review 2005

How well did you know this?
1
Not at all
2
3
4
5
Perfectly
21
Q
21. Motor stimulation of the peroneal nerve elicits what
motions of the foot?
A. Dorsifl exion, inversion
B. Dorsifl exion, eversion
C. Plantar fl exion, inversion
D. Plantar fl exion, eversion
E. Plantar fl exion only
A
  1. Answer: B

Source: Day MR, Board Review 2005

How well did you know this?
1
Not at all
2
3
4
5
Perfectly
22
Q
  1. What is the function of the parasympathetic contribution
    to the bladder?
    A. Causes the detrusor muscle to relax and the sphincter
    to contract
    B. Causes the detrusor muscle to contract and the sphincter
    to relax
    C. Causes the trigone muscle to relax and the sphincter to
    relax
    D. Causes the detrusor muscle to contract and the sphincter
    to contract
    E. Causes the trigone muscle to contractd and the detrusor
    to contract
A
22. Answer: B
Explanation:
Cholinergic tone allows for micturation.
Anticholinergics can cause urinary retention.
Source: Boswell MV, Board Review 2005
How well did you know this?
1
Not at all
2
3
4
5
Perfectly
23
Q
23. Meralgia paresthetica is caused by compression of what
nerve?
A. Obturator
B. Sural
C. Common peroneal
D. Lateral fermoral cutaneous
E. Genitofemoral
A
  1. Answer: D

Source: Day MR, Board Review 2005

How well did you know this?
1
Not at all
2
3
4
5
Perfectly
24
Q
  1. Which of the following most accurately describes the
    pelvic splanchnics nerves?
    A. S3-S4, preganglionic sympatehtic fi bers
    B. S1-S2, postganglionic sympathetic fi bers
    C. S2-S4, preganglionic parasympathetic fi bers
    D. T1-L2/L3, postganglionic sympathetic fi bers
    E. T1-L2/L3, preganglionic parasympathetic fi bers
A
24. Answer: C
Explanation:
C. The pelvic splanchnics are parasympathetic fi bers
arising from the sacral segments.
Source: Boswell MV, Board Review 2005
How well did you know this?
1
Not at all
2
3
4
5
Perfectly
25
Q
25. Which of the following nerves is not a branch of the sciatic
nerve?
A. Superfi cial peroneal
B. Saphenous
C. Deep peroneal
D. Posterior tibial
E. Sural
A
  1. Answer: B

Source: Day MR, Board Review 2005

How well did you know this?
1
Not at all
2
3
4
5
Perfectly
26
Q
  1. A patient is going to undergo a gastrectomy. Where would
    you like the tip of the epidural catheter to be placed?
    A. T3
    B. T5
    C. T10
    D. T12
    E. L1
A
  1. Answer: B
    Explanation:
    (Raj, Pain Review 2nd Ed., page 272)
    With a gastrectomy the incision may be extended up to the
    xiphoid process. This can be extremely painful if the
    catheter is placed to low. Hence T10, T12, L1 catheters may
    be able to cover some of the pain but it would be
    insuffi cient. Higher concentrations and volumes would be
    needed to cover the xiphoid which could put the patient at
    undue risk of hypotension, muscle weakness, and sensory
    loss. A catheter at T3 may be appropriate for thoracic
    surgery but not for upper abdominal surgery.
    Source: Shah RV, Board Review 2005
How well did you know this?
1
Not at all
2
3
4
5
Perfectly
27
Q
  1. Which of the following is true of neural tube
    development?
    A. Closure of the neural tube proceeds in a craniocaudal
    sequence
    B. The basic organization of the neural tube features
    peripheral neuronal cell bodies and centrally located
    myelinated processes
    C. The primitive neurectoderm cells of the neural tube give
    rise to both neuron and all glial components
    D. During development, neuronal and glial precursors are
    born near the central canal and migrate to the periphery
    E. Mature neurons migrate out of the spinal cord to form
    the sensory ganglia
A
  1. Answer: D
    Explanation:
    (Moore, Developing Human, 6/e, pp 452-456.) After
    closure of the neural tube, cells proliferate and establish
    three primitive layers: (1) the ventricular zone adjoining
    the central canal and ventricles; mitoses of neuronal and
    glial precursors continue in this zone; (2) a mantle zone
    consisting of cell bodies of neurons and glia that have
    migrated out of the ventricular zone; and (3) a marginal
    zone on the periphery containing the myelinated nerve
    processes characteristic of white matter. Closure of the
    neural tube begins near the midpoint of its length and
    proceeds in both directions simultaneously. The
    neurectoderm of the neural tube will give rise to neurons
    and some glial cells (astrocytes, oligodendroglia, and
    ependymal cells), but the precursors of microglia (the
    monocyte-macrophage lineage) migrate into the nervous
    system from the blood. The sensory ganglia are formed by
    neural crest cells that migrated before the development of
    mature neurons.
    Source: Klein RM and McKenzie JC 2002.
How well did you know this?
1
Not at all
2
3
4
5
Perfectly
28
Q
28. Which of the following cranial nerves exists the brain stem
from its dorsal aspects?
A. Oculomotor nerve
B. Facial nerve
C. Trigeminal nerve
D. Glossopharyngeal nerve
E. Trochlear nerve
A
  1. Answer: E
    Explanation:
    The trochlear nerve is purely a motor nerve and is the only
    cranial nerve to exist the brain dorsally. The trochlear
    nerve supplies one muscle: the superior oblique. The cell
    bodies that originate in the trochlear nerve are located in
    the ventral part of the brain stem in the trochlear nucleus.
    The trochlear nucleus gives rise to fi bers that cross to the
    other side of the brain stem just prior to exiting the pons.
    Thus, each superior oblique muscle is supplied by nerve
    fi bers from the trochlear nucleus of the opposite side. The
    nerve travels in the lateral wall of the cavernous sinus and
    then enters the orbit via the superior orbital fi ssure. It
    passes medially and diagonally across the levator palpebral
    superioris and superior rectus muscles to innervate the
    superior oblique. (Parent, 531)
    Source: Neurology Examination and Board Review By
    Nizar Souayah, MD and Sami Khella, MD
How well did you know this?
1
Not at all
2
3
4
5
Perfectly
29
Q
  1. The deep peroneal nerve provides cutaneous innervation
    to what part of the foot?
    A. Lateral aspect
    B. The entire dorsum
    C. The plantar surface
    D. The web space between the Great and 2nd toes
    E. The web space between the 3rd and 4th toes
A
  1. Answer: D

Source: Day MR, Board Review 2006

How well did you know this?
1
Not at all
2
3
4
5
Perfectly
30
Q
  1. The sympathetic component of the sphenopalatine
    ganglion travel to the ganglion via what nerve?
    A. Greater petrosal nerve
    B. Palatine nerves
    C. Maxillary nerve
    D. Deep petrosal nerve
    E. Mandibular nerve
A
  1. Answer: D

Source: Day MR, Board Review 2005

How well did you know this?
1
Not at all
2
3
4
5
Perfectly
31
Q
  1. You are asked to insert a needle approximately 2-3 cm
    medial to the ASIS and 2-3 cm inferior to this point. The
    insertion should be above the inguinal ligament. A ‘pop’
    may be felt as you insert your needle to identify the right depth. What nerve are you most likely going to block?
    A. Genitofemoral
    B. Iliohypogastric
    C. Ilioinguinal
    D. Lateral femoral cutaneous
    E. Obturator nerve block
A
  1. Answer: C
    Explanation:
    (Raj, Pain Review 2nd Ed.)
    A.The genitofemoral nerve is approached by inserting the
    needle just lateral to the pubic tubercle. The needle is
    advanced through the inguinal ligament.
    B.The iliohypogastric lies 3 cm medial to the ASIS, but the
    needle is aimed towards the umbilicus.
    C.The ilioinguinal nerve is approximately 2-3 cm medial
    and 2-3 cm inferior to the ASIS, but above the inguinal
    canal.Typically one may feel a ‘pop’ as the needle passes the
    internal oblique and lies between the internal oblique and
    transverses abdominis.
    D.The lateral femoral cutaneous nerve is approached 2cm
    medial and inferior to the ASIS but the needle is inserted
    below the inguinal ligament. The needle is advanced
    through the fascia lata with a pop and loss of resistance.
    Note that if you advance the needle further and feel a
    second pop, then you have gone through the fascia iliaca.
    This is approximately how one would perform a fascia
    iliaca block in order to avoid the femoral nerve block.
    Source: Shah RV, Board Review 2003
How well did you know this?
1
Not at all
2
3
4
5
Perfectly
32
Q
  1. The portion of the upper extremity that is not innervated
    by the brachial plexus is:
    A. Posterior medial portion of the arm
    B. Anterior and posterior aspects of elbow
    C. Lateral portion of the forearm
    D. Medial portion of the forearm
    E. Anterolateral portion of the arm
A
  1. Answer: A
    Explanation:
    The arm receives sensory innervation from the brachial
    plexus except for the shoulder, which is innervated by the
    cervical plexus, and the posterior medial aspect of the arm,
    which is supplied by the intercostobrachial nerve.
How well did you know this?
1
Not at all
2
3
4
5
Perfectly
33
Q
33. The greater occipital nerve is a branch of:
A. Posterior ramus of C2
B. Posterior ramus of C1
C. Anterior ramus of C1
D. Anterior ramus of C2
E. Anterior ramus of C2 and C3
A
  1. Answer: A
    Explanation:
    The skin over the posterior part of the neck, upper back,
    posterior part of the scalp upto the vertex is supplied
    segmentally by the posterior rami of the C2 to C5.
    A. The Greater occipital nerve is a branch of the posterior
    of ramus of C2.
    The lesser occipital nerve is a branch of the posterior
    ramus of C2 and C3.
    Headaches due to occipital neuralgia are characterized
    by either continuous pain or paroxysmal lancinating pain
    in the distribution of the nerve.
    The etiology of occipital neuralgia is compression of
    the C2 nerve root, migraine or nerve entrapment.
    An occipital nerve block maybe performed as a diagnostic
    or therapeutic measure.
    Source: Chopra P. 2004
How well did you know this?
1
Not at all
2
3
4
5
Perfectly
34
Q
34. The muscles of the back receive motor innervation from
A. Dorsal roots
B. Dorsal primary rami
C. Gray rami communicantes
D. Splanchnic nerves
E. Ventral primary rami
A
  1. Answer: B
    Explanation:
    A.The dorsal roots convey sensation to the spinal cord.
    B.The axial musculature of the back receives innervation
    from the dorsal primary rami of the spinal nerves.
    D.The splanchnic nerves and gray rami communicantes
    are components fo the sympathetic division of the
    autonomic nervous sytem.
    E.The ventral primary rami contribute to the cervical
    plexus, brachial plexus, intercostal nerves, and the
    lumbosacral plexus.
How well did you know this?
1
Not at all
2
3
4
5
Perfectly
35
Q
35. Which of the following is the most direct route for spread
of infection from the paranasal sinuses to the cavernous
sinus of the dura mater?
A. Pterygoid venous plexus
B. Parietal emissary vein
C. Frontal emissary vein
D. Basilar venous plexus
E. Superior ophthalmic vein
A
  1. Answer: E
    Explanation:
    A. The pterygoid venous plexus communicates with the
    cavernous sinus via the petrosal sinuses.
    B. The parietal emissary vein also communicates with the
    superior sagittal sinus.
    C. The frontal emissary vein communicates with the
    superior sagittal sinus via the foramen cecum.
    D. The basilar venous plexus communicates with the
    inferior petrosal sinus.
    E. The superior ophthalmic vein drains the region of the
    paranasal sinuses and is directly connected with the
    cavernous sinus although blood fl ow is normally away
    from the brain
How well did you know this?
1
Not at all
2
3
4
5
Perfectly
36
Q
  1. Myelination in the central nervous system differs from
    myelination in the peripheral nervous system in
    A. Its formation only during fetal development
    B. The function of myelin
    C. Its ultrastructural appearance
    D. The involvement of oligodendrocytes
    E. The involvement of astrocytes
A
  1. Answer: D
    Explanation:
    Myelination in the central (CNS) and peripheral (PNS)
    nervous systems occurs by similar methods, although
    there are differences in the supportive cells responsible.
    A. Myelin is similar in both locations but different in the
    presence of Schmidt-Lanterman clefts, which only appear
    in the PNS and represent the presence of Schwann cell
    cytoplasm that is not displaced toward the periphery.
    This provides a continuous cytoplasmic pathway from
    the exterior to the interior of the myelin sheath.
    Myelin is an insulator and also decreases membrane
    capacitance.
    White matter is high in myelin content and is named by
    the presence of tracts of axons that appear white
    (myelinated).
    Gray matter represents neuron-rich areas low in myelin
    (e.g., cell bodies).
    B. Myelination occurs in both pre- and postnatal
    development.
    In the PNS, formation of myelin is initiated by the
    invagination of an axon into a Schwann cell. A mesaxon is
    formed as the outer leafl ets of the cell membrane fuse.
    Subsequently, the mesaxon of the Schwann cell wraps itself
    around the fi ber.
    D. In the CNS, the oligodendrocytes myelinate axons,
    whereas the Schwann cells conduct myelination in the
    PNS.
    Oligodendrocytes myelinate several axons at one time,
    whereas the Schwann cells myelinate only one axon.
    In the CNS, oligodendrocytes form myelin around
    several axon segments compared with the 1: 1 relationship
    between Schwann cells and axon segments in the PNS.
How well did you know this?
1
Not at all
2
3
4
5
Perfectly
37
Q
37. The stellate ganglion lies in closest proximity to which of
the following vascular structures?
A. Common carotid artery
B. Internal carotid artery
C. Vertebral artery
D. Axillary artery
E. Aorta
A
  1. Answer: C
    Explanation:
    The stellate ganglion usually lies in front of the neck of the
    fi rst rib.
    C. The vertebral artery lies anterior to the ganglion as it
    has just originated from the subclavian artery.
    After passing over the ganglion, it enters the foramen
    and lies posterior to the anterior tubercle of C6.
How well did you know this?
1
Not at all
2
3
4
5
Perfectly
38
Q
  1. A cranial fracture through the foramen rotundum that
    compresses the enclosed nerve (maxillary nerve) results
    in the following clinical symptoms:
    A. Inability to clench the jaw fi rmly
    B. Regurgitation of fl uids into the nasopharynx during
    swallowing
    C. Paralysis of the inferior oblique muscle of the orbit
    D. Loss of the sneeze refl ex
    E. Uncontrolled drooling from the mouth
A
  1. Answer: D
    Explanation:
    A. The mandibular division of the trigeminal nerve, which
    passes through the foramen ovale, innervates the
    masticatory muscles responsible for clenching the jaw as
    well as the tensor palatini muscle, which assists in the
    establishment of the velopharyngeal seal.
    B. The other muscles of the soft palate are innervated by
    the pharyngeal branch of the vagus nerve, which transits
    the jugular foramen.
    C. The inferior oblique muscle of the eye is innervated by
    the inferior branch of the oculomotor nerve, which enters
    the orbit through the superior orbital fi ssure.
    D. The maxillary division of the trigeminal nerve, which
    passes through the foramen rotundum, is entirely sensory.
    Damage to this nerve results in sensory deprivation over
    the maxillary region of the face and loss of the sneeze
    refl ex.
    E. The orbicularis oris and buccinator muscles are
    innervated by the facial nerve, which transits the
    stylomastoid foramen
How well did you know this?
1
Not at all
2
3
4
5
Perfectly
39
Q
  1. The occipital portion of the skull receives sensory
    innervation from
    A. Spinal accessory nerve (nerve XI)
    B. Facial nerve (nerve VII)
    C. Ophthalmic branch of trigeminal nerve (nerve V)
    D. Maxillary branch of trigeminal nerve (nerve V)
    E. None of the above
A
  1. Answer: E
    Explanation:
    The occiput receives sensory innervation from the
    occipital nerves, which are terminal branches of the
    cervical plexus.
How well did you know this?
1
Not at all
2
3
4
5
Perfectly
40
Q
  1. For upper abdominal surgery, optimal epidural catheter
    placement would be at which vertebral levels?
    A. C7 - T2
    B. T4 - T8
    C. T8 - T10
    D. T10- T12
    E. L2- L4
A
  1. Answer: C
    Explanation:
    Ref: Crews. Chapter 14. Acute Pain Syndromes. In:
    Practical Management of Pain. 3rd Edition. Raj et al,
    Mosby, 2000, page 178.
    Source: Day MR, Board Review 2003
How well did you know this?
1
Not at all
2
3
4
5
Perfectly
41
Q
  1. The muscles of the anterior compartment of the leg are
    innervated primarily by which of the following nerves?
    A. Deep fi bular
    B. Lateral sural cutaneous
    C. Saphenous
    D. Superfi cial fi bular
    E. Sural
A
  1. Answer: A
    Explanation:
    The common fi bular (peroneal) nerve bifurcates into
    superfi cial and deep branches. The deep fi bular nerve
    innervates all muscles of the anterior compartment of the
    leg. The superfi cial fi bular nerve emerges from the deep
    fascia and descends in the lateral compartment, where it
    innervates the peroneus longus and brevis muscles before
    dividing into median dorsal cutaneous and intermediate
    dorsal cutaneous nerves, which supply the distal third of
    the leg, dorsum of the foot, and all the toes. The saphenous nerve (the terminal branch of the common femoral nerve)
    distributes cutaneous branches to the anterior and medial
    aspects of the leg as well as to the dorsomedial aspect of
    the foot. The sural nerve follows the course of the lesser
    saphenous vein and becomes the lateral sural cutaneous
    nerve to supply the anterolateral aspect of the foot.
    Source: Klein RM and McKenzie JC 2002.
How well did you know this?
1
Not at all
2
3
4
5
Perfectly
42
Q
42. The correct order of structures (from cephalad to caudad)
in the intercostal space is:
A. Nerve, artery, vein
B. Vein, nerve, artery
C. Vein, artery, nerve
D. Artery, nerve, vein
E. Artery, vein, nerve
A
  1. Answer: C
How well did you know this?
1
Not at all
2
3
4
5
Perfectly
43
Q
43. The inner lining of the thoracic cage is also known as the
A. visceral pleura
B. parietal pleura
C. subcostal fascia
D. endothoracic fascia
E. external thoracic fascia
A
  1. Answer: D
    Explanation:
    D. The rib cage is covered both internally and externally
    by thin layers of deep fascia.
    The inner layer, consisting of loose areolar tissue called
    the endothoracic fascia, lines the internal aspect of the
    thoracic cage.
    This layer of facia covers the inner surface of the
    intercostal muscles and intervening ribs, along with
    the subcostal and transversus thoracis muscles andthe
    diaphragm.
    It lies between the parietal pleura and the thoracic cage.
How well did you know this?
1
Not at all
2
3
4
5
Perfectly
44
Q
  1. Which of the following muscles of the larynx is innervated
    by the external branch of the superior laryngeal nerve?
    A. Vocalis muscle
    B. Thyroarytenoid muscles
    C. Posterior cricoarytenoid muscle
    D. Oblique arytenoids muscles
    E. Cricothyroid muscle
A
  1. Answer: E
    Explanation:
    All other muscles of the larynx are innervated by the
    recurrent laryngeal nerve.
How well did you know this?
1
Not at all
2
3
4
5
Perfectly
45
Q
45. The saphenous nerve can blocked at the medial thigh in what named canal?
A. Guyon’s
B. Alcock’s
C. Labat’s
D. Kappis’s
E. Hunter’s
A
  1. Answer: E

Source: Day MR, Board Review 2006

How well did you know this?
1
Not at all
2
3
4
5
Perfectly
46
Q
46. Which of the following has a transverse process but not
vertebral artery foramen?
A. C1
B. C3
C. C5
D. C6
E. C7
A
  1. Answer: E
    Explanation:
    (Bonica, 3rd Ed., page 970)
    C7 has a transverse process but no foramen for
    transmitting the vertebral artery. In fact the stellate
    ganglion (C7-T1) is posterior to the artery at this level.
    C3, C5, C6 all have transverse processes that transmit the
    vertebral artery. C1 does have a long transverse process
    that is longer than other vertebral bodies. It also has a
    vertebral artery foramen. Upon leaving this level, the
    vertebral artery migrates posteriorly and medially.
    Source: Shah RV, Board Review 2005
How well did you know this?
1
Not at all
2
3
4
5
Perfectly
47
Q
47. The Greater splanchnic nerve is formed by?
A. T4 - T6 sympathetic nerve fi bers
B. T5 - T7 sympathetic nerve fi bers
C. T5 - T9 sympathetic nerve fi bers
D. T10 - T11 sympathetic nerve fi bers
E. T11 - T12 sympathetic nerve fi bers
A
  1. Answer: C

Source: Boswell MV, Board Review 2005

How well did you know this?
1
Not at all
2
3
4
5
Perfectly
48
Q
  1. The L4-5 facet joint is innervated by the:
    A. medial branches of the L3 and L4 spinal nerves
    B. medial branches of the L4 and L5 spinal nerves
    C. medial branches of the L2, L3, and L4 spinal nerves
    D. medial branches of the L3, L4, and L5 spinal nerves
    E. medial branches of the L4, L5, and S1 spinal nerve
A
  1. Answer: A
    Explanation:
    (Raj, Pain Review 2nd Ed., page 292-3)
    The medial branch of the exiting spinal nerve sends a
    branch to the facet joint at its level and to one level below.
    In older studies some have demonstrated and ascending
    branch of the dorsal ramus that innervates the level above.
    However, this is not true as far as board exams go.
    In this case the medial branches of the L3 and L4 nerves innervate the L4-5 zygapophyseal joint. Expect some
    permutation of this kind of question
    Source: Shah RV, Board Review 2005
How well did you know this?
1
Not at all
2
3
4
5
Perfectly
49
Q
49. The conus medullaris ends at T12 in which percentage of
patients?
A. 5-10%
B. 15-20%
C. 21-35%
D. Never
E. >50%
A
49. Answer: A
Explanation:
(Bonica, 3rd Ed., page 1480, fi gure 75-9)
The conus ends at:
T12: 6%
T12-L1: 18%
L1: 30%
L1-L2: 24%
L2: 22%
L3: rarely
Source: Shah RV, Board Review 2005
How well did you know this?
1
Not at all
2
3
4
5
Perfectly
50
Q
50. The genitofemoral nerve passes through the anterior psoas
fascia at what vertebral body level?
A. L1
B. L2
C. L3
D. L4
E. L5
A
  1. Answer: C

Source: Day MR, Board Review 2005

How well did you know this?
1
Not at all
2
3
4
5
Perfectly
51
Q
  1. Which of the following is not true about the cervical
    intervertebral disc?
    A. The height of the cervical intervertebral disc is twice
    anteriorly as compared to posteriorly
    B. The vertebral endplates bounding the intervertebral disc
    are fl at
    C. The nucleus pulposus is located anteriorly when compared
    to those in the lumbar spine
    D. The posterior longitudinal ligament is wider when compared
    to its size in the lumbar spine
    E. The joints of Luschka are not synovial joints.
A
  1. Answer: B
    Explanation:
    (Bonica, 3rd Ed., page 372)
    The vertebral endplates are fl at in the lumbar spine, but
    are concave and convex. All of the other statements are
    true.
    The joints of Luschka or the uncovertebral joints are
    degenerative clefts.
    Source: Shah RV, Board Review 2005
How well did you know this?
1
Not at all
2
3
4
5
Perfectly
52
Q
  1. The anterior and posterior spinal arteries originate from
    the
    A. Common carotid and vertebral arteries, respectively
    B. Internal carotid and vertebral arteries, respectively
    C. Internal carotid and posterior cerebral arteries, respectively
    D. Vertebral and anterior cerebellar arteries, respectively
    E. Vertebral and posterior inferior cerebellar arteries, respectively
A
  1. Answer: E
    Explanation:
    The posterior spinal arteries are paired; they arise from the
    posterior inferior cerebellar arteries and have 25 to 40
    radicular arteries. The anterior spinal artery is a single
    midline artery that arises from the union of a branch of
    each vertebral artery. It descends in front of the anterior
    longitudinal sulcus of the spinal cord. This single artery is
    also fed by numerous radicular arteries.
How well did you know this?
1
Not at all
2
3
4
5
Perfectly
53
Q
53. At which level would you expect the spinal canal to be
narrowest in its sagittal dimension?
A. C1-2
B. C2-3
C. C3-4
D. C4-5
E. C5-6
A
53. Answer: E
Explanation:
(Bonica, 3rd Ed., page 975)
Canal sagittal dimensions get progressively narrower
from the upper to lower cervical spine; the narrowest
location is at C5-6.
C1-C3: 21 mm (16-30)
C4-C6: 18 mm (14-23)
Source: Shah RV, Board Review 2005
How well did you know this?
1
Not at all
2
3
4
5
Perfectly
54
Q
  1. The deep peroneal nerve innervates the
    A. Web space between the third and fourth toes
    B. Medial aspect of the dorsum of the foot
    C. Web space between the great toe and the second toe
    D. Entire dorsum of the foot
    E. Lateral aspect of the dorsum of the foot
A
  1. Answer: C
    Explanation:
    The deep peroneal nerve innervates the short extensors
    of the toes and the skin of the web space between the great
    and second toe.
    The deep peroneal nerve is blocked at the ankle by
    infi ltration between the tendons of the anterior tibial and
    extensor hallucis longus muscle.
How well did you know this?
1
Not at all
2
3
4
5
Perfectly
55
Q
55. The medial boundary of the stellate ganglion is?
A. Vertebral artery
B. Dome of the lung
C. Longus colli muscle
D. Scalene muscles
E. Subclavian artery
A
  1. Answer: C

Source: Day MR, Board Review 2005

How well did you know this?
1
Not at all
2
3
4
5
Perfectly
56
Q
56. The hypogastric plexus is composed of what type of
fi bers?
A. Postganglionic sympathetic
B. Postganglionic parasympathetic
C. Visceral efferent
D. A delta
E. C Fibers
A
  1. Answer: A
    Source: Raj P, Pain medicine - A comprehensive Review -
    Second Edition
How well did you know this?
1
Not at all
2
3
4
5
Perfectly
57
Q
  1. The odontoid process (dens) is correctly described by
    which of the following statements?
    A. It articulates with the occipital portion of the skull
    B. It is separated from the atlas by an intervertebral disk
    C. It projects from the inferior surface of the atlas
    D. It represents the vertebral body of the fi rst cervical vertebra
    E. None of the above
A
  1. Answer: D
    Explanation:
    D. The odontoid process (dens) of the axis, the second
    cervical vertebra, is the remnant of the body of the fi rst
    cervical vertebra (atlas).
    Developing from a separate ossifi cation center, it fused
    to the body of the axis.
    The fact that there is no intervertebral disk between the
    atlas and axis probably facilitates the fusion.
    The dens, projecting from the superior surface of the
    axis, provides a pivot about which rotation occurs at the
    atlantoaxial joint.
    Fracture and posterior dislocation of the dens may crush
    the spinal cord with fatal results.
How well did you know this?
1
Not at all
2
3
4
5
Perfectly
58
Q
58. What is the rib that articulates with the sternum at the
level of the xiphisternal junction?
A. 6th
B. 7th
C. 8th
D. 9th
E. 10th
A
  1. Answer: B
    Explanation:
    B.The 7th costal cartilage articulates with the sternum at
    the lateral margin of the xiphisternal junction.
    The articulation between the xiphoid process and the
    inferior border of the sternal body is a cartilaginous
    structure that ossifi es later in life.
How well did you know this?
1
Not at all
2
3
4
5
Perfectly
59
Q
  1. The following statement is false regarding the course of the
    vertebral artery:
    A. The vertebral artery is consistently lateral to the atlantoaxial
    joint.
    B. The vertebral artery courses through the foramina transversaria
    of C7 through C1.
    C. The vertebral artery lies posterior to the atlanto-occipital
    joint in the suboccipital triangle
    D. Intracranial branches from the right and left vertebral
    arteries combine to form a single descending anterior
    spinal artery.
    E. The paired right and left posterior spinal arteries originate
    respectively from the right and left vertebral arteries
    above C1.
A
  1. Answer: B
    Explanation:
    Reference: Gray’s Anatomy, Thirteenth American Edition.
    Pages 696-699.
    The vertebral arteries originate from the subclavian
    arteries and ascend anterior to the transverse processes of
    the seventh cervical vertebrae. They then ascend through
    the foramina transversaria from C6 to C1. The C7
    vertebral body does not have a foramen transversarium.
    The arteries then exit the C1 foramina transversaria and
    turn medial to course through the foramen magnum
    where they ascend and combine to form the basilar artery
    which perfuses the brain. Just above the foramen magnum,
    the vertebral arteries give rise to the paired posterior
    spinal arteries. Further cephalad, at the approximate level
    of the medulla oblongata, the paired vertebrals give rise to
    the descending, single anterior spinal artery.
    The vertebral arteries are always lateral to the atlanto-axial
    joints since they must pass through the laterally-situated
    foramina transversaria at C1 and C2. From the foramen
    transversarium of C1 to the foramen magnum, the
    vertebral arteries pass posterior to the atlanto-occipital
    joints within the suboccipital triangle. There course from
    C1 to the foramen magnum may be somewhat tortuous
    and unpredictable.
    The vertebral arteries are always anterolateral with respect to the cervical neuroforamina since they are coursing
    through the foramina transversaria.
    Source: Schultz D, Board Review 2004
How well did you know this?
1
Not at all
2
3
4
5
Perfectly
60
Q
60. The sympathetic component to the sphenopalatine
ganglion originates from which nerve?
A. Deep petrosal
B. Greater petrosal
C. Maxillary
D. Greater palatine
E. Lesser palatine
A
  1. Answer: A

Source: Day MR, Board Review 2006

How well did you know this?
1
Not at all
2
3
4
5
Perfectly
61
Q
61. At which level is the intervertebral disc height to vertebral
body height the largest?
A. C1-C2
B. C6-C7
C. T12-L1
D. L4-L5
E. C3-C4
A
  1. Answer: B
    Explanation:
    (Bonica 3rd Ed., page 971)
How well did you know this?
1
Not at all
2
3
4
5
Perfectly
62
Q
62. A musculocutaneous nerve lesion affects
A. Hand sensation
B. Supination with the forearm extended
C. Supination with the elbow in fl exion
D. Wrist extension
E. Upper arm abduction
A
  1. Answer: C
    Explanation:
    The musculocutaneous nerve arises from the lateral cord
    of the brachial plexus and carries fi bers from the root of
    C5, C6, and C7. The nerve proceeds obliquely downward
    between the axillary artery and the median nerve. The
    nerve pierces the coracobrachialis muscle while giving off
    branches to it, and it descends further between the biceps
    and brachialis muscles to supply both of them. The lateral
    cutaneous nerve of the forearm is the sensor continuation
    of the muscular cutaneous nerve innervates the skin from
    the elbow to the wrist and covers the entire forearm from
    the dorsal to the ventral midline. The coracobrachialis
    muscle is a forward elevator of the arm. The biceps is a
    forearm supinator, especially if the elbow fl exed at 90
    degrees. Isolated lesions of the musculocutaneous nerve
    are rare. Such lesion could cause weakness of elbow
    fl exion again resistance in a fully supinated hand, possible
    arm elevation weakness, arm pain and radial forearm
    parasthesia. (Brazis, 9-10; Staal, 31-33)
    Source: Neurology Examination and Board Review By
    Nizar Souayah, MD and Sami Khella, MD
How well did you know this?
1
Not at all
2
3
4
5
Perfectly
63
Q
63. Corneal anesthesia results from blockade of which nerve?
A. Supratrochlear
B. Mandibular
C. Maxillary
D. Opthalmic
E. Meckels
A
  1. Answer: D

Source: Day MR, Board Review 2006

How well did you know this?
1
Not at all
2
3
4
5
Perfectly
64
Q
64. The styloid process is an important landmark during
blockade of which nerve?
A. phrenic nerve
B. maxillary nerve
C. facial nerve
D. glossopharyngeal nerve
E. trigeminal nerve
A
  1. Answer: D
    Explanation:
    D. The tip of the styloid process lies approximately
    halfway between the angle of the mandible and the
    mastoid process and provides a bony landmark for
    blockade of the glossopharyngeal nerve.
    The glossopharyngeal nerve exits the jugular foramen at
    the base of the skull to emerge slightly posterior and
    medial to the styloid process. It proceeds inferiorly to
    innervate the posterior one-third of the tongue as well as
    part of the throat and nasopharynx as far down as the
    pharyngoesophageal junction at the level of the cricoid
    cartilage.
How well did you know this?
1
Not at all
2
3
4
5
Perfectly
65
Q
65. There is usually no sympathetic ganglion at:
A. L1
B. L2
C. L3
D. L4
E. L5
A
  1. Answer: A

Source: Racz G. Board Review 2003

How well did you know this?
1
Not at all
2
3
4
5
Perfectly
66
Q
  1. Blood supply to the spinal cord is by:
    A. Two posterior spinal arteries and two anterior spinal
    arteries
    B. Two posterior spinal arteries and one anterior spinal
    artery
    C. Branches of the lumbar arteries
    D. Radicularis Magna (artery of Adamkiewicz) and two
    posterior spinal arteries
    E. Braches of Aorta
A
  1. Answer: B
    Explanation:
    The blood supply to the spinal cord is primarily three
    longitudinally running arteries – two posterior spinal
    arteries and one anterior spinal artery.
    The Anterior Spinal artery supplies approximately 80% of
    the intrinsic spinal cord vasculature. It is formed by the
    union of a branch from the terminal part of each vertebral
    artery. It actually consists of longitudinal series of
    functionally individual blood vessels with wide variation
    in lumen size and anatomic discontinuations.
    The spinal cord has three major arterial supply regions:
  2. C1 to T3: Cervicothoracic region
  3. T3 to T8: Mid thoracic region
  4. T8 to the Conus: Thoracolumbar region
    There is a poor anastomosis between these three regions.
    As a result the blood fl ow at the T3 and T8 levels is
    tenuous. In spinal stenosis, especially in the lower cervical
    region, the Anterior Spinal artery may be compressed by a
    dorsal osteophyte and a herniated nucleus pulposus
    leadingto the Anterior Spinal Syndrome (loss of motor
    function).
    There are two posterior spinal arteries that arise from the
    posterior inferior cerebellar arteries.
    The three longitudinal arteries are reinforced by ‘feeder’
    arteries. They are spinal branches of the cervical, vertebral
    posterior intercostal, lumbar and lateral sacral arteries.
    Approximately 6 or 7 of these contribute to the anterior
    spinal artery and another 6 or 7 to the posterior spinal
    arteries, but at different levels. The largest of these arteries
    is known as the radicularis magna or the artery of
    Adamkiewicz.
    Source: Chopra P, 2004
How well did you know this?
1
Not at all
2
3
4
5
Perfectly
67
Q
  1. Which of the following neurological structures does NOT
    travel through the cavernous sinus
    A. Sympathetic carotid plexus
    B. Oculomotor nerve
    C. Mandibular branch of the trigeminal nerve
    D. Trochlear nerve
    E. Abducens nerve
A
  1. Answer: C
    Explanation:
    The medial wall of the cavernous sinus contains the
    abducens nerve, the internal carotid artery and the
    sympathetic fi bers of the carotid plexus. The lateral wall
    contains the oculomotor and trochlear nerves, and the
    ophthalmic and maxillary divisions of the trigeminal
    nerve. (Afi fi and Bergman, 240)
    Source: Neurology Examination and Board Review By
    Nizar Souayah, MD and Sami Khella, MD
How well did you know this?
1
Not at all
2
3
4
5
Perfectly
68
Q
  1. Which of the following is TRUE about the trigeminal
    nerve?
    A. The spinal nucleus of the trigeminal nerve subserves
    light touch in the ipsilateral side of the face.
    B. The motor nucleus of the trigeminal nerve lies in the
    pons medial to the sensory nucleus and sends axons to
    the maxillary division of the trigeminal nerve
    C. The three divisions of the trigeminal nerve converge at
    the Gasserian ganglion.
    D. The mesencephalic nucleus of the trigeminal nerve
    subserves pain and temperature in the ipsilateral side
    of the face.
    E. The mandibular division of the trigeminal nerve subserves
    sensation of the ipsilateral angle of the mandible.
A
  1. Answer: C
    Explanation:
    The trigeminal nerve is a mixed nerve. It subserves the
    sensory innervation of the ipsilateral side of the face and
    the ipsilateral muscles of mastication (masseter, temporalis, and pterygoids). The sensory nucleus of the
    trigeminal nerve extends from the midbrain to the upper
    cervical cord: (a) The mesencephalic nucleus subserves
    proprioception and deep sensation from the tendons and
    muscles of mastication. (b) The main sensory nucleus
    (located in the pons) subserves light touch. (c) The spinal
    nucleus (which extends from the pons to the upper
    cervical cord and is divided into segments that correspond
    to concentric dermatomes around the mouth) subserves
    pain and temperature. The trigeminal nerve supplies
    sensation to the ipsilateral side of the face via three
    branches: the ophthalmic division (which innervates the
    frontal, lacrimal, and nasociliary areas), the maxillary
    division (which innervates the cheek and lower eyelid),
    and the mandibular division (which innervates the lower
    lip, the tongue, the mandible, except for the angle of the
    mandible). The motor nucleus lies medially to the main
    sensory nucleus and sends axons to the mandibular
    division of the trigeminal nerve. All division of the
    trigeminal nerve converge at the Gasserian ganglion which
    lies in Meckel’s cave of the temporal bone. (Afi fi and
    Bergman, 173-175)
    Source: Neurology Examination and Board Review By
    Nizar Souayah, MD and Sami Khella, MD
How well did you know this?
1
Not at all
2
3
4
5
Perfectly
69
Q
69. All of the following muscles are innervated by the medial
division of the sciatic nerve EXCEPT the
A. Semimembranosus
B. Long head of the biceps femoris
C. Semitendinosus
D. Short head of the biceps femoris
E. Adductor magnus muscle
A
  1. Answer: D
    Explanation:
    The sciatic nerve is a mixed nerve that carries fi bers from
    L4 to S3 and leaves the pelvis through the sciatic foramen
    below the piriform muscle. The nerve then curves laterally
    and downward beneath gluteus maximus muscle and runs
    on the dorsal side of the femoral bone to terminate at the
    proximal part of the popliteal fossa to divide into the tibial
    nerve medially and the peroneal nerve laterally. Within
    the sciatic nerve, as proximal as the gluteal region, the
    fi bers of the tibial and peroneal nerves are arranged into
    two separate divisions: The medial and the lateral trunks,
    respectively. The medial part of the nerve innervates the
    adductor magnus and the hamstring muscles, except for
    the short head of the biceps femoris (it is the only thigh
    muscle supplied by the lateral peroneal division). The
    hamstring muscles are fl exors of the knee joint and include
    the semimembranosus muscle, the semitendinous muscle,
    and the short and long heads of the biceps femoris. (Staal,
    117-118)
    Source: Neurology Examination and Board Review By
    Nizar Souayah, MD and Sami Khella, MD
How well did you know this?
1
Not at all
2
3
4
5
Perfectly
70
Q
  1. The thoracic duct ascends from the abdominal cavity to
    enter the thorax via the following aperture:
    A. Esophageal
    B. Central tendon
    C. Splanchnic
    D. Inferior vena caval
    E. Aortic
A
  1. Answer: E
    Explanation:
    The diaphragm has three large openings (the aortic,
    esophageal, and vena caval apertures) and a number of
    smaller ones that transmit the superior and middle
    splanchnic nerves.
    E. The aortic aperture is the lowest and most posterior of
    the large openings and approximates the level of the T10
    vertebra.
    This aperture also transmits the thoracic duct and occasionally the azygous and hemiazygous veins.
How well did you know this?
1
Not at all
2
3
4
5
Perfectly
71
Q
71. Celiac plexus is located:
A. in front of vena cava at L2
B. in front of aorta at L2
C. in front of aorta at L1
D. behind the vena cava of L1
E. behind the aorta of L1
A
  1. Answer: C

Source: Racz G. Board Review 2003

How well did you know this?
1
Not at all
2
3
4
5
Perfectly
72
Q
  1. Age-related changes which occur in the spine and may be
    imaged by MRI fi ndings include:
    A. Increase in water content of intervertebral disk
    B. Increase in glycoproteins of intervertebral disk
    C. Increase in height of vertebral bodies
    D. Reduced caliber of spinal canal
    E. All of the above
A
  1. Answer: D
    Explanation:
    With advancing age, there is reduced caliber of the spinal
    canal due to arthritic changes. There is decreased water
    and glycoprotein content of intervertebral disks, with
    decreased height of vertebral bodies. (Ref. 1, pp. 455–456;
    Ref. 2, p. 590)
    Source: Neurology for the Psychiatry specialty Board
    Review By Leon A. Weisberg, MD
How well did you know this?
1
Not at all
2
3
4
5
Perfectly
73
Q
73. Which of the following structures receives afferents
responsible for taste sensation in the anterior two thirds
of the tongue ?
A. Submaxillary ganglion
B. Pterygopalatine ganglion
C. Superior salivary nucleus
D. Geniculate ganglion
E. Submandibular ganglion
A
  1. Answer: D
    Explanation:
    The nervus intermedius is the sensory and
    parasympathetic division of the facial nerve. It carries
    preganglionic parasympathetic fi bers to the submaxillary
    ganglion and tothe pterygopalatine ganglion. It receives
    sensory fi bers from the geniculate ganglion. This ganglion
    receives fi bers that carry taste sensation from the anterior
    two-thirds of the tongue and afferents from the mucosae
    of the pharynx, nose, and palate.(Afi fi and Bergman, 166-
    167)
    Source: Neurology Examination and Board Review By
    Nizar Souayah, MD and Sami Khella, MD
How well did you know this?
1
Not at all
2
3
4
5
Perfectly
74
Q
74. Which of the following cranial nerves is responsible for
eye closure ?
A. Oculomotor nerve
B. Trochlear nerve
C. Abducens nerve
D. Facial nerve
E. Spinal accessory nerve
A
  1. Answer: D
    Explanation:
    The orbicularis oculi controls eye closure and is
    innervated by the facial nerve.Eye opening is controlled by
    the levator of the lid, which is innervated by the
    oculomotor nerve. (Brazis, Masdeu, and Biller, 271-272)
    Source: Neurology Examination and Board Review By
    Nizar Souayah, MD and Sami Khella, MD
How well did you know this?
1
Not at all
2
3
4
5
Perfectly
75
Q
  1. Which statement is false regarding the blood supply of the
    spinal cord:
    A. The anterior spinal artery originates from the vertebral
    arteries above the foramen magnum.
    B. Segmental spinal arteries enter the spinal canal by way
    of the intervertebral foramina bilaterally at every spinal
    level.
    C. The blood supply to the spinal cord is most tenuous in
    the region from T10 through L1.
    D. The Artery of Adamkiewicz is an anterior medullary
    feeder artery that contributes blood to the anterior
    spinal artery.
    E. Arteries that penetrate the cord parenchyma are end
    arteries and usually do not anastomose further.
A
  1. Answer: C
    Explanation:
    Reference: Gray’s Anatomy, Thirteenth American Edition.
    Pages 964-971.
    The spinal cord receives its’ blood supply from three
    longitudinal arteries:
  2. a single anterior spinal artery
  3. two posterior spinal arteries
    The anterior spinal artery forms intracranially anterior to
    the medulla oblongata, from the junction of two anterior
    spinal branches, one derived from each of the two
    vertebral arteries. From its’ origin, it descends anterior
    to the spinalcord to the tip of the conus medullaris. The
    diameter of the anterior spinal artery is greatest at the
    cervical and lower thoracic regions with the smallest
    diameter along the midthoracic zone from T3-T9.
    This region of the cord is considered the “vulnerable zone”
    with respect to circulation.
    The branches of the anterior spinal artery that
    penetrate the cord parenchyma are end arteries and do not anastomose further.
    The spinal cord receives segmental arteries bilaterally at
    every level that enter the spinal canal through the
    neuroforamina, accompanying the spinal nerve roots.
    These segmental arteries supply blood to the dorsal and
    ventral nerve roots. In the cervical region these segmental
    arteries may originate from the vertebrals or from other
    cervical arteries. In the thoracic region, the segmental
    arteries originate from the posterior intercostal arteries
    which branch directly from the aorta.In the lumbar
    region,they branch from the lumbar arteries. In addition,
    the anterior spinal artery is reinforced at a number of
    segmental levels by feeder arterial branches from these
    segmental arteries. These arteries are called anterior
    medullary feeder arteries. There is an average total of 8
    anterior medullary feeder arteries (inclusive of all spinal
    levels bilaterally) the largest of which is the great anterior
    medullary artery or artery of Adamkiewicz. The total
    number of anterior medullary feeder arteries varies from 2
    to 17 in different individuals with an average of 3 in the
    cervical region, 3 in the thoracic region and 2 in the
    lumbar region. The artery of Adamkiewicz typically enters
    the cord on the left side (77% of specimens) anywhere
    from T7 to L4 (most commonly at T9 to T12). In the
    cervical region, the largest anterior medullary feeder
    enters at C4-5 or C5-6.
    Source: Schultz D, Board Review 2004
How well did you know this?
1
Not at all
2
3
4
5
Perfectly
76
Q
  1. A 40-year-old man developed chronic pain in the right
    forearm that lasted hours each day. Neurological
    examination demonstrated normal sensory examination,
    mild right forearm pronation weakness, and weak fl exion
    of terminal phalanges of right thumb, index, and middle
    fi ngers. An attempt to make a full circle by applying the
    end phalanx of the thumb to that of the index fi nger with
    fi rm pressure showed consistent weakness. Which of the
    following structures is affected?
    A. Right anterior interosseous nerve
    B. Right median nerve at the upper axilla
    C. Right ulnar nerve
    D. Right radial nerve
    E. Right musculocutaneous nerve
A
  1. Answer: A
    Explanation:
    The patient described in this vignette has a pure motor
    defi cit. The right pronator quadratus is weak because of
    paresis of forearm pronation. Also there is paresis of the
    fl exor digitorum profundus I & II and the fl exor pollicis
    longus because of loss of fl exion of the terminal phalanges
    of the second and third fi ngers, and the thumb,
    respectively. All of these muscles are innervated by an
    anterior interosseous nerve. The characteristic feature of a
    lesion of this nerve is the inability to make a circle with the
    thumb and index fi nger. (Staal 55-56)
    Source: Neurology Examination and Board Review By
    Nizar Souayah, MD and Sami Khella, MD
How well did you know this?
1
Not at all
2
3
4
5
Perfectly
77
Q
77. The mandibular nerve leaves the cranial cavity through
the
A. foramen ovale
B. foramen spinosum
C. foramen rotundum
D. jugular foramen
E. foramen lacerum
A
  1. Answer: A
    Explanation:
    A. The madibular nerve emerges from the cranial cavity by
    way of the foramen ovale to enter the infratemporal fossa.
    In the infratemporal fossa the mandibular nerve divides
    into its terminal branches.
    Madibular nerver supplies the lower jaw, tongue, and
    lower teeth, the buccal surface of the cheek, and the
    skin overlying the lower jaw, the temporal region, and the
    anterosuperior two-thirds of the surface of the external
    ear.
    C. The maxillary nerve leaves through the foramen
    rotundum
How well did you know this?
1
Not at all
2
3
4
5
Perfectly
78
Q
78. The most frequently involved structure in external
impingement of shoulder is
A. Subacromiodeltoid tendon
B. Subacromiodeltoid bursa
C. Teres minor tendon
D. Infraspinatus tendon
E. Supraspinous tendon
A
  1. Answer: B

Source: Sizer et al - Pain Practice - March & June 2004

How well did you know this?
1
Not at all
2
3
4
5
Perfectly
79
Q
  1. Anatomy of Meckel’s cave is as follows.
    A. the location of the glossophraryngeal nerve as it passes
    near the tonsillar fossa
    B. an intestinal diverticulum that may cause epigastric
    pain
    C. located between the mastoid process and the angle of the
    mandiblethe retrouterine recess in which an abscess or
    tumor may compress the hypogastric plexus
    D. the recess in which the gasserian ganglion resides before
    dividing
    E. the recess in which the posteior two thirds of the ganglion
    are covered by dura
A
  1. Answer: E
    Explanation:
    The gasserian ganglion, also known as the trigeminal
    ganglion,is formed from many midpontine rootlets as they
    pass into the posterior cranial fossa and cross the superior
    border of the petrous bone to enter the recess called
    Meckel’s cave, or the trigeminal cave.
    In this recess (Meckel’s cave or trigeminal cave), the
    posterior two thirds of the ganglion are covered by dura.
    The anterior one-third is not covered by dura, and it is
    from this portion of the ganglion that the three major
    divisions of the trigeminal nerve (ophthalmic, maxillary,
    and mandibular) exit.
    When performing a trigeminal ganglion block, if the
    needle has punctured the dura of Meckel’s cave, a very
    small amount of local anesthetic may result in rapid loss
    of consciousness or cardiac arrest.
How well did you know this?
1
Not at all
2
3
4
5
Perfectly
80
Q
80. The highest concentration of wide dynamic range neurons
are seen in Rexed lamina
A. I
B. II
C. III
D. V
E. IX
A
  1. Answer: D
    Explanation:
    The Rexed laminae in the dorsal horn of the spinal
    cord are important in the modulation and transmission
    of nociceptive stimuli.
    A, B. Laminae I and II receive dense projections from the
    brainstem nuclei.
    Small diameter primary afferents also terminate
    primarily in laminae I and II.
    Myelinated primary afferent neurons and those from
    the brainstem involved in modulation of pain perception
    in laminae I and II also project to the deeper laminae such
    as V.
    D. Lamina V has a high concentration of wide dynamic
    range cells, which play an active role in the phenomenon
    of central sensitization.
How well did you know this?
1
Not at all
2
3
4
5
Perfectly
81
Q
  1. The L2 ganglion is at the following distance from the
    anterior lateral x-ray view- vertebral body border:
    A. 5-6 MM
    B. 6-8 MM
    C. 12-13 MM
    D. 15-20 MM
    E. 20-22 MM
A
  1. Answer: C

Source: Racz G. Board Review 2003

How well did you know this?
1
Not at all
2
3
4
5
Perfectly
82
Q
82. Cutaneous innervation of the plantar surface of the foot is
provided by the
A. Sural nerve
B. Posterior tibial nerve
C. Saphenous nerve
D. Deep peroneal nerve
E. Superfi cial peroneal nerve
A
  1. Answer: B
    Explanation:
    There are 5 nerves that supply the ankle and foot:
  2. Posterior tibial nerve
  3. Sural nerve
  4. Superfi cial nerve
  5. Deep peroneal nerve
  6. Saphenous nerve
    These nerves are superfi cial at the level of the ankle and
    are easy to block. The posterior branch of the tibial nerve
    gives rise to the medial and lateral plantar nerves, which supply the plantar surface of the foot.
How well did you know this?
1
Not at all
2
3
4
5
Perfectly
83
Q
83. While performing an atlantoaxial joint injection, the
patient has a seizure. Local anesthetic injection into what
structure/space most likely occurred?
A. Carotid artery
B. Epidural space
C. Intrathecal space
D. Spinal nerve root
E. Vertebral artery
A
  1. Answer: E

Source: Day MR, Board Review 2006

84
Q
84. Which anatomic landmark is not used in any of the
standard blocks of the sciatic nerve?
A. Posterior superior iliac spine (PSIS)
B. Anterior superior iliac spine (ASIS)
C. Greater trochanter of the femur
D. Pubic tubercle
E. Lesser trochanter of the femur
A
  1. Answer: E

Source: Shah RV, Board Review 2003

85
Q
85. The nerve located immediately lateral to the trachea is:
A. Vagus
B. Recurrent laryngeal
C. Phrenic
D. Long thoracic
E. Spinal accessory
A
  1. Answer: B
    Explanation:
    The structures in the neck from medial to lateral are the
    recurrent laryngeal nerve, carotid artery, vagus nerve,
    internal jugular vein, and phrenic nerve.
86
Q
86. The branches of the spinal artery providing blood supply
to the vertebral body is:
A. Posterior
B. Intermediate
C. Lateral
D. Medial
E. Anterior
A
  1. Answer: E
    Explanation:
    The spinal artery arises from the posterior branch of the
    segmental artery close to the intervertebral foramen and
    divides into three terminal branches: posterior,
    intermediate, and anterior.
    A. The posterior branches help supply the spinal dura and
    the tissues of the epidural space.
    B. The intermediate (middle) branches supply the dura of
    the associated nerve roots.Their radicular branch can
    pierce the dura and help supply the spinal cord.
    E. The anterior branches supply the vertebral bodies along
    with other spinal structures.
87
Q
  1. The artery of Adamkiewicz, also known as arteria
    radicularis magna, implied in spinal cord damage
    following epidural steroid injections, most frequently
    arises from the aorta, the following spinal level(s).
    A. T1 to T4
    B. T5 to T8
    C. L1 to L4
    D. T9 to T12
    E. L5-S1
A
  1. Answer: D
    Explanation:
    The artery of Adamkiewicz, also known as arteria
    radicularis magna, is one of the feeder arteries for the
    anterior spinal artery. Damage to this artery by any means
    or particular injection can lead to ischemia in the
    thoracolumbar region of the spinal cord. The origin of
    this artery is variable. In 60% of the cases, it is described to
    arise from T9 to T12. In 14% of cases, it is described to
    originate from T5 to T8. In 20% of the cases, it is
    described to originate below L1.
88
Q
88. The lesser occipital nerve is formed from the ventral root/s
of which cervical spinal nerve/s?
A. C1
B. C2
C. C3
D. C1 and C2
E. C2 and C3
A
  1. Answer: E

Source: Day MR, Board Review 2006

89
Q
89. What other cranial nerve can be blocked while performing
a glossopharyngeal nerve block?
A. Trigeminal
B. Spinal Accessory
C. Vestibulocochlear
D. Oculomotor
E. Facial
A
  1. Answer: B

Source: Day MR, Board Review 2005

90
Q
90. Which of the following is not part of a Horner’s
syndrome?
A. Myopia
B. Facial anhidrosis
C. Myosis
D. Enophthalmus
E. Ptosis
A
  1. Answer: A

Source: Day MR, Board Review 2006

91
Q
91. Which of the following pairs of cranial nerves travel
through the internal auditory canal?
A. Vestibulocochlear and trigeminal
B. Facial and trigeminal
C. Facial and optic
D. Facial and vestibulocochlear
E. Vestibulocochlear and vagus
A
  1. Answer: D
    Explanation:
    The facial nerve leaves the pons and travels with the
    vestibulocochlear nerve through the internal auditory
    canal. (Parent, 154-168)
    Source: Neurology Examination and Board Review By
    Nizar Souayah, MD and Sami Khella, MD
92
Q
92. Which of the following structures is found in the lateral
wall of the tonsillar fossa?
A. Facial nerve
B. Glossopharyngeal nerve
C. Hypoglossal nerve
D. Lingual nerve
E. Vagus nerve
A
  1. Answer: B
    Explanation:
    A. The facial nerve lies superfi cial on the face.
    B. The location of the glossopharyngeal nerve in the
    tonsillar bed places it in jeopardy during tonsillectomy.
    C, D. The hypoglossal and lingual nerves pass well
    inferior to the tonsillar bed.
93
Q
93. The most medial structures in the antecubital fossa
include:
A. Brachial artery
B. Cephalic vein
C. Tendon of the biceps
D. Median nerve
E. Musculocutaneous nerve
A
  1. Answer: D
94
Q
94. The maxillary nerve leaves the cranial cavity thru the
A. foramen ovale
B. foramen spinosum
C. foramen rotundum
D. jugular foramen
E. foramen lacerum
A
  1. Answer: C
    Explanation:
    A. The mandibular nerve leaves through the foramen
    ovale.
    B.The maxillary nerve leaves the cranial cavity through the
    foramen rotundum.Following here it traverses the
    pterygomaxillary fossa to enter the fl oor of the orbit at the
    inferior orbital fi ssure.
    The maxillary nerve primarily supplies the upper jaw,
    lateral nasal wall, and most of the nasal septum.
95
Q
  1. The saphenous nerve at the ankle is blocked where?
    A. Anterior to the lateral malleolus
    B. Posterior to the lateral malleolus
    C. Anterior to the medial malleolus
    D. Posterior to medial malleolus
    E. Lateral to the extensor hallucis longus muscle
A
  1. Answer: C

Source: Day MR, Board Review 2006

96
Q
96. If a needle is introduced 2 cm inferior and lateral to
the pubic tubercle, to which nerve will it lie in close
proximity?
A. Ilioinguinal nerve
B. Femoral nerve
C. Lateral femoral cutaneous nerve
D. Obturator nerve
E. Sciatic nerve
A
  1. Answer: D
    Explanation:
    D. An obturator nerve block is achieved by placement of
    the needle 1 to 2 cm lateral to and below the pubic
    tubercle.
    After contact with the pubic bone, the needle is withdrawn
    and walked cephalad to identify the obturator canal.
97
Q
97. The L3 ganglion located on lateral view X-ray:
A. inferior anterior border of L3
B. mid body L3
C. posterior body L3
D. near the anterior border of L3
E. none of the above
A
  1. Answer: D

Source: Racz G. Board Review 2003

98
Q
  1. In carpal tunnel syndrome, the median nerve is entrapped
    A. Beneath the fl exor retinaculum ligament
    B. Above the fl exor retinaculum ligament
    C. At the hamate bone
    D. In Guyon’s canal
    E. On the radial side of the wrist at the level of the styloid
    process
A
  1. Answer: A
    Explanation:
    The point of entrapment of the median nerve in carpal
    tunnel syndrome lies under the fl exor retinaculum. The
    fl exor retinaculum forms the roof of the carpal tunnel,
    whereas the carpal bones and their connective tissue
    components form the fl oor of the carpal tunnel.In Guyon’s
    canal, the hamate, and the pisiform bones are sites of
    compression of the ulnar nerve at the wrist. Rarely, radial
    nerve compression occurs at the level of the styloid
    process, just proximal to the wrist. (Staal, 56-66)
    Source: Neurology Examination and Board Review By
    Nizar Souayah, MD and Sami Khella, MD
99
Q
  1. The obturator nerve innervates muscles which are
    responsible for what movement of the lower extremity?
    A. Abduction
    B. Flexion
    C. Extension
    D. Internal rotation
    E. Adduction
A
  1. Answer: E

Source: Day MR, Board Review 2006

100
Q
  1. Which of the following is a compression site of the radial
    nerve?
    A. Suprascapular notch
    B. Carpal tunnel
    C. Spinoglenoid notch
    D. The elbow posterior to the medial epicondyle
    E. Spiral groove in the posterior aspect of the humerus
A
  1. Answer: E
    Explanation:
    The radial nerve arises from the posterior cord of the
    brachial plexus and comprises fi ber from spinal levels C5
    to C8. After descending posterior to the axillary artery, the
    nerve courses posterior to the humerus in the spinal
    groove. It is at this site that the nerve is more often
    damaged by the compression. (Staal, 35)
    Source: Neurology Examination and Board Review By
    Nizar Souayah, MD and Sami Khella, MD
101
Q
  1. Blockade of the ilioinguinal nerve is accomplished by
    depositing local anesthetic between which 2 muscle
    groups?
    A. External oblique and internal oblique
    B. External oblique and transverse abdominus
    C. Rectus abdominus and internal oblique
    D. Internal oblique and transverse abdominus
    E. Transverse abdominus and iliacus
A
  1. Answer: D

Source: Day MR, Board Review 2006

102
Q
102. Which brachial plexus block technique produces the best blockade of all of the terminal branches of the plexus?
A. Axillary
B. Supraclavicular
C. Infraclavicular
D. Interscalene
E. Deep cervical plexus
A
  1. Answer: B

Source: Day MR, Board Review 2006

103
Q

103.Which one of the following statements regarding
zygapophyseal joints is false:
A. Zygapophyseal joints are covered with a fi brous capsule
and contain synovial fl uid
B. The articular surfaces of zygapophyseal joints are covered
with hyaline cartilage.
C. The most cephalad zygapophyseal joint is C1-2.
D. Zygapophyseal joints are always posterior spinal structures.
E. Lumbar zygapophyseal joints are oriented with increasing
alignment to the sagittal plane as one ascends the
spine

A
  1. Answer: C
    Explanation:
    References:
    Waldman, Interventional Pain Management, Second
    Edition; Chapter 42, pp. 446-452
    Gray’s Anatomy, Thirteenth American Edition, pp. 333-
    335.
    Zygapophyseal or facet joints are true synovial joints and
    share characteristics with other synovial joints including:
    1.The contiguous bony surfaces of the joints are covered
    with hyaline cartilage
    2.The joint is surrounded by a fi brous joint capsule
    3.The inner surface of the joint is lined by a synovial
    membrane which secretes synovial fl uid to lubricate the
    joint
    4.The joints contain intra-articular meniscoids (of
    uncertain signifi cance) which are attached to the joint
    capsule
    Zygapophyseal joints are posterior spinal structures with
    the nerve root and neuroforamen anterior to the joint. The
    most cephalad zyg joint is C2-3 and the most caudal is L5-
    S1. The atlanto-axial and atlanto-occipital joints (C1-2
    and C0-1 respectively) are unique and fundamentally
    different from zyg joints in that they are anterior spinal
    structures with the nerve root exiting posterior to the
    joint. These two most cephalad spinal joints are therefore
    not considered zygapophyseal joints and do not have
    sensory innervation from medial branch nerves like the
    true zyg joints.
    Source: Schultz D, Board Review 2004
104
Q
104. The long thoracic nerve innervates the
A. Serratus anterior muscle
B. Rhomboid muscle
C. Levator scapula
D. Supraspinatus muscle
E. Infraspinatus muscle
A
  1. Answer: A
    Explanation:
    The long thoracic nerve arises from the motor roots of C5,
    C6, and C7. It courses downward through and in front of
    the medial scalenus muscle and further descends dorsal to
    the brachial plexus along the medial axillary wall to
    innervate the serratus anterior muscle. The suprascapular
    nerve innervates the supraspinatus and infraspinatus. The
    dorsal scapular nerve innervates the rhomboid and levator
    scapulae. (Parent, 276; Staal 19)
    SOURCE: Souayah, N, and Khella S; Neurology
    Examination & Board Review; McGraw-Hill, New York.
    Source: Neurology Examination and Board Review By
    Nizar Souayah, MD and Sami Khella, MD
105
Q
105. Which of the following is a branch of the trigeminal
nerve:
A. Lingual nerve
B. Posterior auricular nerve
C. Spinal accessory nerve
D. Greater Occipital nerve
E. Lesser Occipital nerve
A
  1. Answer: A
    Explanation:
    The lingual nerve is a branch of the mandibular division.
    The posterior auricular is part of the cervical plexus, the
    spinal accessory is CN XI, and the occipital is from C2 and
    C3
    Source: Trescot AM, Board Review 2003
106
Q

106.The arachnoid villi allow cerebrospinal fl uid to pass
between which of the following two spaces?
A. Choroid plexus and subdural space
B. Subarachnoid space and subdural space
C. Superior sagittal sinus and jugular vein
D. Subdural space and cavernous sinus
E. Subarachnoid space and superior sagittal sinus

A
  1. Answer: E
    Explanation:
    E. Cerebrospinal fl uid formed in the choroid plexus
    circulates in the subarachnoid space and is absorbed by the
    venous sinuses through the arachnoid villi, some of which
    project into the superior sagittal sinus.
    Cerebrospinal fl uid protects the nervous system from
    concussions and mechanical injuries and is important for
    metabolism.
    A, B, C, D. It circulates slowly through the ventricles of the
    brain and through the meshes of the subarachnoid space.
107
Q
  1. The MOST common location of the Dorsal Root Ganglion
    is:
    A. Medial to the pedicle within the lateral recess
    B. Inferolateral to the pedicle
    C. Lateral to the superior articular facet of the corresponding
    vertebra.
    D. Directly below the pedicle
    E. Superolateral to the pedicle
A
  1. Answer: D
    Explanation:
    In approximately 90% of cases the dorsal root ganglion
    (DRG) lies in the middle zone of the intervertebral
    foramen, directly below the pedicle. In approximately, 8%
    of cases it is inferolateral and in 2% of cases it is medial to
    the pedicle. The center of the DRG lies over the lateral
    portion of the intervertebral disc in some cases. The size of
    the DRG increases from L1 to S1 and then progressively
    decreases till S4. The DRG at S1 is 6mm in width.
    The DRG contains multiple sensory cell bodies. It is the
    site for production of neuropeptides: Substance P,
    Eukephalin, VIP (Vasoactive Intestinal peptides), and
    other neuropeptides.
    The DRG is a primary source of pain when it undergoes
    mechanical deformity as by an osteophyte, herniated
    nucleus pulposus or stenosis. It also produces pain when it
    undergoes an infl ammatory process either by infection or
    chemical irritation from a herniated nucleus pulposus,
    release of local neuropeptides or local vascular
    compromise.
    Source: Chopra P, 2004
108
Q
  1. A pulse in the dorsalis pedis artery may be palpated
    A. Between the tendons of the extensor digitorum longus
    and peroneus tertius muscles
    B. Between the tendons of the extensor hallucis and extensor
    digitorum muscles
    C. Between the tendons of the tibialis anterior and extensor
    hallucis longus muscles
    D. Immediately anterior to the lateral malleolus
    E. Immediately posterior to the medial malleolus
A
  1. Answer: B
    Explanation:
    B. The dorsal pedal artery, a continuation of the anterior
    tibial artery, passes onto the dorsum of the foot between
    the tendons of the extensor hallucis longus and extensor
    digitorum longus muscles.
    The dorsal pedal pulse may be palpated here before the
    artery passes beneath the extensor hallucis brevis muscle.
    E. The posterior tibial artery passes behind the medial
    malleolus, where the posterior tibial pulse is normally
    palpable.
    Source: Klein RM and McKenzie JC 2002.
109
Q
  1. In the histogenesis of the neural tube, which zone will
    become the white matter of the adult CNS?
    A. Ventricular zone
    B. Marginal zone
    C. Mantle zone
    D. Ependymal zone
    E. Intermediate zone
A
  1. Answer: B
    Explanation:
    A. From the ventricular zone, astrocytes, oligodendrocytes,
    and neurons differentiate.
    Ultimately, the cells that remain in the ventricular zone
    become the ependymal cells that line the central canal.
    B. The white matter of the adult CNS is derived from the
    marginal zone of the developing neural tube.
    The most peripheral zone is the marginal zone, which
    contains the myelinated axons of the developing motor
    neurons (adult white matter).
    C. The mantle zone forms the grey matter, where cell
    bodies of differentiating motor neurons are located.
    Source: Klein RM and McKenzie JC 2002.
110
Q
  1. Which of the following is INCORRECT about the axillary
    nerve?
    A. It arises from C5-C6.
    B. It innervates the deltoid and teres minor muscles.
    C. It is a pure motor never.
    D. Its injury may lead to weakness of arm abduction in the
    horizontal position.
    E. In neuralgic amyotrophy, the axillary nerve may be affected
    in isolation in 10% of cases.
A
  1. Answer: C
    Explanation:
    The axillary nerve originates from the posterior fascicle of
    the brachial plexus and carries fi bers from C5 and C6. It
    innervates the deltoid muscle and teres minor muscle. The
    axillary nerve sends a sensory branch, the lateral brachial
    cutaneous nerve, to the skin of the upper outer surface of
    the arm mainly in the deltoid region. An axillary nerve
    lesion results in weakness of arm abduction in the
    horizontal position against resistance.The fi rst 30 degrees
    of abduction of the upper arm from the trunk is
    performed by the supraspinatus muscle, which is
    innervated by the suprascapular nerve, not by the axillary
    nerve.
    There is also weakness of the horizontal upper arm
    retraction against resistance with sensory loss in the skin
    area overlying the deltoid muscle.The axillary nerve
    is often involved in neuralgic amyotrophy, and in about
    ten percentof cases it is affected in isolation. (Parent, 275
    – 277; Staal, 27-29)
    Source: Neurology Examination and Board Review By
    Nizar Souayah, MD and Sami Khella, MD
111
Q
111. The venous sinuses are located in the
A. Periosteum
B. Dura mater
C. Subdural space
D. Arachnoid
E. Pia mater
A
  1. Answer: B
    Explanation:
    A. The periosteum is an important connective tissue layer
    surrounding the bone of the skull. This layer retains
    osteogenic potential even in the adult.
    B. The dura mater contains the venous sinuses and is
    composed of dense connective tissue and possesses very
    limited osteogenic potential.
    The dura mater is one of the three protective layers that
    comprise the meninges surrounding the brain and spinal
    cord.
    In the spinal cord, the dura is separated from the
    periosteum by the epidural space.
    C.The thin subdural space lies between the dura mater and
    the arachnoid.
    D. The arachnoid is composed of a weblike avascular
    connective tissue that forms villi for the reabsorption of
    cerebrospinal fl uid (CSF) into the venous sinuses found in
    the dura.
    The subarachnoid space contains the CSF, which is
    formed both by ultrafi ltration of the blood and
    transport across the epithelial lining of the choroid plexuses.
    E. The pia covers the brain and spinal cord as a delicate,
    vascular connective tissue.
    It lines the perivascular spaces through which blood
    vessels penetrate the CNS.
112
Q
112. Innervation to the rotator cuff muscle that medially
rotates the arm is provided by the
A. Axillary nerve
B. Suprascapular nerve
C. Thoracodorsal nerve
D. Upper and lower subscapular nerves
E. None of the above
A
  1. Answer: D
    Explanation:
    (Moore, Anatomy, 4/e, pp 698-699.) The upper and lower
    subscapular nerves innervate the subscapularis muscle,
    which is the only muscle of the rotator cuff group that
    medially rotates the arm. The lower subscapular nerve also
    innervates the teres major muscle, which is not part of the
    rotator cuff group. The suprascapular nerve innervates the
    supraspinatus and infraspinatus muscles that abduct and
    laterally rotate the arm, respectively. The teres minor
    muscle, innervated by the axillary nerve, also laterally
    rotates the arm. The thoracodorsal nerve, originating from
    the posterior cord between the upper and lower
    subscapular nerves, innervates the latissimus dorsi muscle.
    Source: Klein RM and McKenzie JC 2002.
113
Q
  1. The vertebral arteries are correctly described by which of
    the following statements?
    A. They arise from the common carotid artery on the left
    and the brachiocephalic artery on the right
    B. They enter the cranium via the anterior condylar canals
    C. They enter the cranium via the posterior condylar canals
    D. They pass through the transverse foraminae of several
    cervical vertebrae
    E. They directly give rise to the posterior cerebral arteries
A
  1. Answer: D
    Explanation:
    A, D. The vertebral arteries usually arise from the
    subclavian arteries and ascend through the transverse
    foramina of the sixth to the fi rst cervical vertebrae but not
    the seventh.
    They enter the cranium through the foramen magnum
    after which they join to form the basilar artery.
    B. The hypoglossal nerves leave the cranium via the
    anterior condylar (hypoglossal) canals,
    C. The posterior condylar canals transmit emissary veins.
    E. The basilar artery terminates by bifurcating into the
    posterior cerebral arteries.
114
Q
114. Areas innervated by glossopharyngeal nerve are all of the following EXCEPT
A. Palatine tonsils
B. Posterior one third of the tongue
C. Pharyngeal wall
D. Epiglottis
E. Auditory Canal
A
  1. Answer: D
    Source: Raj P, Pain medicine - A comprehensive Review -
    Second Edition
115
Q
115. Nutrition to the lumbar intervertebral disk is from the:
A. Posterior Spinal artery
B. Internal Iliac artery
C. Lumbar artery
D. Anterior spinal artery
E. None of the above
A
  1. Answer: C
    Explanation:
    The lumbar arteries supply the vertebrae at various levels.
    Each lumbar artery passes posteriorly around the related
    vertebra and supplies branches into the vertebral body.
    The terminal branches form a plexus of capillaries below
    each endplate. The disk is a relatively avascular structure.
    Nutrition to the disk is by diffusion from the endplate
    capillaries and blood vessels in the outer annulus fi brosus.
    Passive diffusion of fl uids into the proteoglycan matrix is
    further enhanced by repeated compression of the disk by
    repeated fl exion-extension of the spine associated with
    activities of daily living which pumps fl uid in and out of
    the disk.
    Source: Chopra P. 2004
116
Q
116. The carotid tubercle (Chaussignac’s tubercle) is located
at:
A. Transverse process of the C5 vertebra
B. Facet joint of the C5-C6 vertebra
C. Facet joint of the C6-C7 vertebra
D. Transverse process of the C7 vertebra
E. Transverse process of the C6 vertebra
A
  1. Answer: E
    Explanation:
    E. The carotid tubercle (Chaussignac’s tubercle) lies 2.5
    cm lateral to the cricoid cartilage.
    It is a part of the transverse process of the C6 vertebra
    and can be easily palpated anteriorly.
    The carotid tubercle is an important landmark for stellate
    ganglion blocks.
    Source: Chopra P. 2004
117
Q
117. The MOST common origin of the Artery of Adamkiewicz
is:
A. Between T4 and T6
B. At T7
C. Between T8 and L3
D. At L4
E. At L5
A
  1. Answer: C
    Explanation:
    A, B. It originates in 14% of cases between T5-T8.
    In a small percentage of cases (15%) the take off is
    higher at T5. In this case the slender contribution from the
    iliac artery enlarges to compensate for the increased blood
    fl ow to the lumbar portion of the cord and the conus.
    C. The artery of Adamkiewicz originates on the left
    between the T8 and L3 level in most cases.
    This is the largest of the feeder arteries that supplies the
    anterior spinal artery.
    The artery of Adamkiewicz enters through an
    intervertebral foramen between T8 and L3 to supply the
    lumbar enlargement. It originates in 60% of cases between
    T9-T12.
    D, E. It originates below L1 in 20% of cases.
    Source: Chopra P, 2004
118
Q
  1. The Stellate ganglion is located at the:
    A. Anterior to the transverse process of the C6 vertebra
    B. Posterior to the subclavian artery
    C. Anterior to the transverse process of the C5 vertebra
    D. Anterior to the neck of the fi rst rib and the transverse
    process of the C7 vertebra
    E. Anterior to vertebral artery at C7
A
  1. Answer: D
    Explanation:
    D. The stellate ganglion is the inferior cervical ganglion.
    The cervicothoracic ganglion is frequently formed by
    the fusion of the inferior cervical ganglion and the fi rst
    thoracic ganglion.
    It is located anteriorly on the neck of the fi rst rib and
    the transverse process of the C7 vertebra.
    It is oval in shape and 1 inch long by 0.5 inch wide.
    The ganglion is bound anteriorly by the subclavian
    artery, posteriorly by the prevertebral fascia and the
    transverse process, medially by the longus colli
    muscle and laterally by the scalene muscle.
    The classical Stellate ganglion block is done one level
    above the location of the Stellate ganglion (it lies at the C7
    level and the block is done at the C6 level).
    E. The vertebral artery travels anteriorly over the stellate
    ganglion at C7 but at C6 the artery moves posteriorly.
    Source: Chopra P. 2004
119
Q
  1. The blood-brain barrier is formed by
    A. Fenestrations between brain capillary endothelial cells
    B. Microglial activity
    C. Astrocytic foot processes surrounding blood vessels entering
    the brain parenchyma
    D. The basement membrane associated with the glia limitans
    E. Occluding junctions between brain capillary endothelial
    cells
A
  1. Answer: E
    Explanation:
    A. The capillary endothelium is nonfenestrated, which also
    adds to the barrier.
    B. Microglia function as brain macrophages and are involved in antigen presentation and phagocytosis.
    C. Astrocytes form foot processes around the brain
    capillaries.
    D. Surrounding the CNS is a basement membrane with a
    lining of astrocyte foot processes; this forms the glia
    limitans, which also contributes to the integrity of the
    blood-brain barrier.
    E. The blood-brain barrier is formed primarily by
    occluding junctions (zonulae occludentes) between
    endothelial cells that compose the lining of brain
    capillaries.
120
Q
  1. Nerves that originate from the sacral plexus include
  2. Femoral nerve
  3. Obturator nerve
  4. Lateral femoral cutaneous nerve
  5. Sciatic nerve
A
  1. Answer: D (4 Only)
    Explanation:
  2. The femoral nerve originates from lumbar plexus.
  3. The obturator nerve originates from lumbar plexus.
  4. Lateral femoral cutaneous nerves arise from the lumbar
    plexus.
  5. The sciatic nerve gives multiple branches: the common
    peroneal deep and superfi cial peroneal, posterior tibial,
    and sural.
121
Q
  1. The greater occipital nerve:
  2. Is a branch of the posterior ramus of C2
  3. Is part of the cervical plexus.
  4. Injection can cause a total spinal.
  5. Is a mixed motor and sensory nerve.
A
  1. Answer: B (1 & 3)
    Explanation:
  2. The greater occipital nerve is a branch of the posterior
    ramus of C2.
  3. The lesser occipital nerve is part of the cervical plexus.
  4. Total spinal anesthetics has occurred after occipital
    nerve blocks, due to subarachnoid injection.
  5. The occipital nerve is a pure sensory nerve.
    Source: Trescot AM, Board Review 2003
122
Q
  1. Which of the following statements are true regarding
    medial branch nerves:
  2. The medial branches from C4 through C7 typically lack
    any cutaneous distribution.
  3. The medial branch of L5 is located in the groove between
    the sacral ala and the superior articulating process
    of S1.
  4. The “third occipital nerve” originates from the C3 dorsal
    ramus.
  5. The greater occipital nerve is the medial branch of the
    dorsal primary ramus of C2.
A
  1. Answer: E (All)
    Explanation:
    References:
    Bogduk, Clinical Anatomy of the Lumbar Spine and
    Sacrum, Third Edition, Churchill Livingston 1997 pp.
    133-135
    Lord, Barnsley, Bogduk; Cervical Zygapophyseal Joint Pain
    in Whiplash Injuries in Cervical Extension-Flexion
    Whiplash Injuries, George Malanga MD Editor, Hanley
    and Belfus Medical Publishers, 1998 pp. 309-310
    With respect to the zygapophyseal joint, each joint is
    innervated by two medial branch nerves, one originating
    from the dorsal ramus of the same segmental level and one
    originating from the dorsal ramus above the level. For
    example, the C4-5 zygapophyseal joint is innervated by the
    C4 and C5 medial branches and the L4-5 zygapophyseal
    joint is innervated by the L3 and the L4 medial branches.
    With respect to the medial branch nerves, each medial branch originates from the dorsal ramus of the spinal
    nerve and sends an ascending branch to innervate the joint
    above and a descending branch to innervate the joint
    below.
    Which medial branch nerves innervate which
    zygapophyseal joints can be a source of confusion since
    numbering in the cervical spine is different from the
    numbering in the thoraco-lumbar spine. The numbering
    convention associating spinal nerve roots within
    neuroforamina is different above and below C7-T1. The
    C4 nerve root (and C4 medial branch) exits the C3-4
    neuroforamen whereas the L4 nerve root (and L4 medial
    branch) exits the L4-5 neuroforamen. Things change from
    a numbering perspective at C7-T1 since the C8 nerve root
    exits the C7-T1 foramen Above C7-T1 (from C2-3 to C6-
    7)the exiting spinal nerve root has the same number as the
    last number of the foramen (i.e. C6 exits the C5-6
    foramen).Beginning at the T1-2 neuroforamen, the exiting
    nerve root carries the same number as the fi rst number of
    the foramen (T1 nerve root exits T1-2 foramen). With
    respect to medial branches, the C8 medial branch
    innervates the C7-T1 joint and the T1-2 joint. The T1
    medial branch innervates the T1-2 and the T2-3 joints.
    And the L3 nerve root innervates the L3-4 and L4-5 joints.
  2. The medial branches of C4 through C8 typically lack
    cutaneous distribution and follow the following course:
    The dorsal ramus of the segmental nerve at each level
    gives rise to the medial branch with that segmental
    number which in turn innervates the zyg joint above and
    below i.e. the C4 spinal nerve exits the C3-4 foramen and
    gives rise to the C4 medial branch which sends a branch
    cephalad to innervate the C3-4 zyg joint and caudad to
    innervate the C4-5 zyg joint.
  3. The medial branch of L5 is located in the groove
    between the sacral ala and the superior articulating process
    of S1.
  4. The C3 medial branch is the fi rst clinically signifi cant
    medial branch with respect to radiofrequency denervation
    of the zyg joints. The C3 medial branch is unique in that it
    has a large superfi cial, cephalad branch called the “third
    occipital nerve”and a smallerdeep and more caudal branch
    which is called the medial branch of C3. The third
    occipital nerve originates from the C3 dorsal ramus and
    courses directly posterior across the lateral aspect of the
    C2-3 zyg joint which it innervates. The C3 medial branch
    also originates from the C3 dorsal ramus but courses
    caudally to cross the lateral waist of the C3
  5. The medial branch of C2 is large and is called the
    greater occipital nerve. The C2 medial branch (greater
    occipital nerve) supplies sensory-motor innervation to the
    occiput and may also send a communicating branch
    caudally to contribute to the innervation of the C2-3 zyg
    joint. articular pillar in similar fashion to the courses
    of the C4-C8 medial branches. The C3 medial branch
    innervates the superior aspect of the C3-4 zyg joint.
    Source: Schultz D, Board Review 2004
123
Q
123. The tibial nerve is responsible for what motion of the foot
and ankle?
1. Plantar fl exion
2. Eversion
3. Inversion
4. Dorsifl exion
A
  1. Answer: B

Source: Day MR, Board Review 2006

124
Q
124. Bony landmarks that need to be identifi ed for a posterior
sciatic nerve block are:
1. Anterior superior iliac spine
2. Posterior superior iliac spine
3. Lesser trochanter
4. Greater trochanter
A
  1. Answer: C

Source: Day MR, Board Review 2006

125
Q
  1. All the following are true statements concerning
    intervertebral disks.
  2. in the lumbar region, the disks constitute 50 percent of
    the length of the column
  3. the nucleus pulposus is a colloidal gel composed of
    mucopolysaccharide
  4. at birth, an intervertebral disk contains 50 percent
    water
  5. the superior and inferior plates of the disk are composed
    of hyaline cartilage
A
  1. Answer: C (2 & 4)
    Explanation:
  2. In the lumbar region the intervertebral disks constitute
    about 30 percent of the length of the column as compared
    with 20 to 25 percent in the thoracic and cervical regions.
  3. Each intervertebral disk is composed of a tough
    fi brocartilaginous ring (the annulus fi brosus) and a
    pliable intergelatinous mass (the nucleus pulposus).
    The colloidal gel of the nucleus pulposus is a
    mucopolysaccharide that can imbibe external fl uid and
    maintain its intrinsic water balance.
  4. At birth the disk contains 88 percent water, but it
    dehydrates with age and trauma.
  5. The superior and inferior plates of the disks are the end
    plates of the vertebral bodies, which are composed of
    articular hyaline cartilage in direct contact with and
    adherent to the underlying resilient bone of the vertebral
    body.
126
Q
  1. Except for the fi rst intercostal nerve, all intercostal nerves
    differ from other spinal nerves in that
  2. each pursues an independent course
  3. they only have a sensory modality
  4. they do not result in the formation plexus
  5. their posterior divisions only supply muscles and skin
    of the back
A
  1. Answer: B (1 & 3)
    Explanation:
  2. Intercostal nerves differ from other spinal nerves in that
    each pursues an independent course.
  3. The intercostal nerves are distributed chiefl y to the
    thorax and abdomen.
  4. Except for the fi rst intercostal nerve, they do not enter
    into the formation of plexuses.
  5. The smaller posterior primary divisions diverge from
    their anterior counterparts and run posteriorly to supply
    the muscles and skin of the back through medial and
    lateral branches.
127
Q
127. The extraocular muscles innervated by the oculomotor
nerve include the
1. lateral rectus
2. medial rectus
3. superior oblique
4. inferior rectus
A
  1. Answer: C (2 & 4)
    Explanation:
    Six extraocular muscles control the movements of the eye.
    The four rectus muscles (superior, medial, inferior, and
    lateral) originate from a common tendon ring that
    encircles the optic foramen.
  2. The lateral rectus muscle is innervated by the abducens
    or 6th cranial nerve while the superior oblique muscle
    receives innervation from the trochlear or 4 th cranial
    nerve.
  3. The oculomotor or third cranial nerve innervates the superior, medial, and inferior rectus muscles as well as the
    inferior oblique and levator palpebrae superioris muscles.
  4. The superior oblique muscle originates above and
    medial to the optic foramen, while the inferior oblique
    muscle originates medially from the periosteum of the
    lacrimal bone.
  5. The oculomotor or third cranial nerve innervates the
    superior, medial, and inferior rectus muscles as well as the
    inferior oblique and levator palpebrae superioris muscles.
128
Q
  1. Referred pain to the penis can be caused by which of the following neuralgias?
  2. Ilioinguinal
  3. Iliohypogastric
  4. Genitofemoral
  5. Lateral femoral cutaneous
A
  1. Answer: A (1, 2, & 3)
    Explanation:
  2. Pain referred to the penis may be due to neuralgia of the
    ilioinguinal nerve.
  3. Pain referred to penis may be due to neuralgia of
    iliohypogastric nerve.
  4. Pain referred to penis may be due to neuralgia of
    genitofemoral nerve.
  5. Lateral femoral cutaneous neuralgia (meralgia
    paresthetica) usually causes pain in the lateral thigh
    without radiation to the penile shaft.
    Source: Kahn CH, DeSio JM. PreTest Self Assessment and
    Review. Pain Management. New York, McGraw-Hill, Inc.,
    1996.
129
Q
  1. The musculocutaneous nerve innervates which muscle/s?
  2. Biceps brachii
  3. Brachialis
  4. Coracobrachialis
  5. Brachiradialis
A
  1. Answer: A

Source: Day MR, Board Review 2006

130
Q
  1. The following are true statements about the sympathetic
    nervous system:
  2. Most cell bodies of preganglionic fi bers lie in the intermediolateral
    cell column of the spinal cord from T1 to
    L2.
  3. The preganglionic fi bers enter the dorsal root ganglion
    through the white rami communicantes.
  4. The preganglionic fi bers enter the spinal nerve through
    the ventral root.
  5. The post ganglionic fi bers exit the sympathetic ganglion
    via the gray rami communicantes
A
  1. Answer: E (All)
    Explanation:
    Reference:
    Bonica’s Management of Pain, Third Edition, pp. 210-215
    Cell bodies of preganglionic sympathetic fi bers lie
    primarily in the intermediolateral cell column of the spinal
    cord from T1 to L2. The preganglionic axon fi bers course
    out of the spinal cord in the ventral root, course a short
    distance within the segmental spinal nerve and then exit
    the spinal nerve via the white rami communicantes. The
    white rami carries the preganglionic sympathetic fi bers to
    the sympathetic ganglia which consist of postganglionic
    cell bodies and are located in the prevertebral regions from
    the high cervical spine to the sacrum. Preganglionic fi bers
    may ascend or descend within the sympathetic chain for
    several levels before synapsing in the ganglion with the
    cell body of the post-ganglionic fi bers.
    The postganglionic sympathetic axons are then
    transmitted out of the ganglion by the gray rami
    communicantes where these axons travel with the spinal
    nerves to their end organs mainly blood vessels, sweat
    glands and hair follicles.
    Source: Schultz D, Board Review 2004
131
Q
  1. Regarding a myelinated nerve fi ber, which of the following
    statements are true
  2. the action potential is regenerated only at the nodes of
    Ranvier
  3. the resting potential is about - 90 microvolts (μV)
  4. sodium channels are present only at the nodes of Ranvier
  5. potassium ions fl ow inward across the cell membrane
    during depolarization
A
  1. Answer: A (1, 2, & 3 )
    Explanation:
    An action potential occurs when an electrical, mechanical,
    or chemical stimulus increases neural membrane
    permeability to ion infl ux.
  2. In myelinated nerves, the action potentials occur at the
    nodes of Ranvier and impulses are rapidly conducted by
    saltatory conduction.
    - In nonmyelinated nerves, the action potentials occur
    along the length of the axon. Such continuous conduction
    is much slower than the saltatory conduction of
    myelinated fi bers.
  3. The resting transmembrane potential is around -90μV
    with high membrane permeability to potassium (K+) and
    limited sodium (Na+) permeability.
  4. Sodium channels are present only at the nodes of
    Ranvier.
  5. Increasing stimuli alter the membrane potential, the
    membrane becomes much more permeable to Na+. This
    causes sodium to fl ow in and potassium ions to fl ow
    outward. Chloride ion diffuses freely through the
    membrane in response to changes in polarity.
    - The opening and closing of K+ channels in response to
    stimuli is responsible for the depolarization and
    repolarization phases of the action potential. An ion
    channel, for example, enlarges during the depolarization
    phase (+60 to 70 μV). After depolarization, the Na+
    channels lose their increased permeability to Na+, and K+
    is pumped back into the cells during repolarization. The
    potential gradually returns to resting transmembrane
    potential.
    - The resting potential is maintained by diffusion of
    intracellular K+ out of the cell through partially open K+
    channels.
132
Q
  1. The sphenopalatine ganglion:
  2. Has sensory input from the trigeminal nerve
  3. Is a parasympathetic ganglion
  4. Has been injected to treat cluster headaches
  5. When injected, causes a Horner’s syndrome
A
  1. Answer: B (1 & 3 )
    Explanation:
  2. The sphenopalatine ganglion receives sensory input
    from a variety of nerves, including the trigeminal nerve.
  3. It is a sympathetic ganglion.
  4. Has been blocked to treat cluster headache.
  5. It has efferents to the stellate ganglion, but does not
    cause a Horner’s syndrome.
    Source: Trescot AM, Board Review 2003
133
Q

133.The following statements are true concerning
intervertebral discs:
1. The structure of the posterior annulus is similar in lumbar
and cervical discs.
2. The annulus of the intact lumbar disc consists of 10 to
20 lamella made up of vertically and horizontally oriented
collagen fi bers.
3. In the intact lumbar disc, the annulus is thickest in the
posterior portion and thinnest in the anterior portion
4. The gray rami commicantes innervates the outer third
of the anterior and anterolateral annulus.

A
  1. Answer: D (4 Only)
    Explanation:
    References:
    Mercer, The Ligaments and Anulus Fibrosus of Human
    Adult Cervical Intervertebral Discs, SPINE 1999;24:619
    Bogduk, Clinical Anatomy of the Lumbar Spine and
    Sacrum, Third Edition, Churchill Livingston 1997,
    Chapter 2, pp. 26-29
  2. The lumbar intervertebral discs function as load bearing shock absorbers whereas the cervical discs
    accommodate rotational motion of the cervical spine but
    do not bear signifi cant weight.
    - Therefore the anatomic structure of the lumbar and
    cervical discs is very different from each other.
  3. The lumbar disc annulus consists of approximately 20
    concentric lamellae of collagen fi bers that uniformly
    surround the nucleus pulposus with alternating oblique
    orientations slanted to approximately 65 degrees.
    - These lamellae are thicker in the anterior and lateral
    portions of the disc and thinner at the posterior disc
    margin.
  4. The cervical anulus is crescentic, being thick anteriorly
    but tapering in thickness laterally as it approaches the
    uncovertebral region. There is no defi nable nucleus in the
    adult cervical disc. Posteriorly, the anulus fi brosus is not
    multilaminated like the anterior anulus of cervical discs or
    the posterior anulus of lumbar discs, nor does it consist of
    obliquely orientated fi bers. It is represented only by a thin
    set of vertically running fi bers.
  5. In the lumbar disc, the gray rami communicantes
    innervates the anterior and anterolateral portions of the
    disc whereas the sinuvertebral nerve innervates the
    posterior disc annulus
    Source: Schultz D, Board Review 2004
134
Q

134.Which of the following tendons are considered
intra-articular but extrasynovial in proximity of the
glenohumeral joint?
1. Infraspinatus Tendon
2. Subscapularis Tendon
3. Supraspinatus Tendon
4. Biceps Tendon

A
  1. Answer: D (4 Only)

Source: Sizer Et Al - Pain Practice March & June 2003

135
Q
  1. Approaches to the brachial plexus block include:
  2. Interscalene
  3. Supraclavicular
  4. Infraclavicular
  5. Cervical transforaminal at C6 level
A
  1. Answer: A (1, 2, & 3)
    Explanation:
  2. The brachial plexus may be blocked through
    interscalene approach.
  3. The brachial plexus may be blocked through
    supraclavicular approach.
  4. The brachial plexus may be blocked through
    infraclavicular or axillary approach.
  5. Cervical transforaminal is utilized for epidural block –
    individual nerves may be blocked, but not the plexus.
    Source: Trescot AM, Board Review 2003
136
Q
  1. Ankle blocks typically refer to blockade of the distal
    branches of which of the following nerves?
  2. Common Peroneal
  3. Tibial
  4. Femoral
  5. Obturator
A
  1. Answer: A (1, 2, & 3 )
    Explanation:
    (Raj, Pain Review 2nd Ed.)
    Ankle blocks typically refer to blockade of the deep and
    superfi cial peroneal nerves, posterior tibial nerves, sural
    nerve, and saphenous nerve.
    The superfi cial peroneal, sural, saphenous are superfi cial
    to the fascia, whereas the deep peroneal and posterior
    tibial are deep to the fascia.
    The superfi cial peroneal and deep peroneal nerves are branches of the common peroneal nerve.
    The deep peroneal is medial to the dorsalis pedis artery.
    The posterior tibial nerve, a branch of the tibial nerve, lies
    in the tarsal tunnel, anterior to the posterior tibial artery
    and posterior to the fl exor hallicis longus. The saphenous
    nerve is a branch of the femoral nerve. The obturator
    supplies the hip adductors and sensation to a small area
    over the distal medial thigh.
    Source: Shah RV, Board Review 2003
137
Q

137.True statements regarding the ligamentum fl avum
include
1. it is thinnest in the cervical region
2. it connects the laminate of adjacent vertebrae
3. it is thickest in the lumbar region
4. its fi bers are arranged parallel to the laminae to which
it is attached

A
  1. Answer: A (1, 2, & 3 )
    Explanation:
    Each ligamentum fl avum consists of yellow elastic tissue
    attached to the anterior and inferior surfaces of the lamina
    above and to the posterior superior surface of the lamina
    below.
    1 & 3. Ligamenta fl ava is thin in the cervical region,
    thicker in the thoracic region, and thickest in the lumbar
    region.
  2. The ligamenta fl ava connect the laminae of adjacent
    vertebrae.
  3. The fi bers of the ligamenta fl ava are oriented
    perpendicularly to the laminae to which they are attached.
138
Q
  1. Which of the following are true regarding a Chiari II
    malformation?
  2. Most common serious posterior fossa malformation
  3. Typically associated with myelomeningocele
  4. Known as Arnold Chiari malformation
  5. Associate with an alpha-fetoprotein marker
A
  1. Answer: E (All)
    Explanation:
    The Chiari II malformation is a complex anomaly with
    skull, dura, brain, spinal, and spinal cord manifestations.
    This disorder is almost invariably associated with
    myelomeningocele. The Chiari II malformation is the
    most common serious malformation of the posterior
    fossa. The frequency is approximately 1 case per 1000
    population in the United States.
    Source: Boswell MV, Board Review 2004
139
Q
  1. Ganglion impar is:
  2. lowest sympathetic ganglion in the body
  3. also named ganglion of Walther
  4. located at sacrococcygeal junction
  5. has ganglion cells
A
  1. Answer: E (All)

Source: Racz G. Board Review 2003

140
Q
140. Head and neck nerves that are mixed (sensory and motor)
nerves include:
1. Glossopha.ryngeal
2. Occipital
3. Sphenopalatine
4. Spinal accessory
A
  1. Answer: D (4 Only)
    Explanation:
    The glossopharyngeal and sphenopalatine are mixed
    nerves. The spinal accessory is a motor nerve and the
    occipital is a sensory nerve.
    Source: Trescot AM, Board Review 2003
141
Q
  1. The posterior cord gives rise to what nerve/s?
  2. Median
  3. Axillary
  4. Ulnar
  5. Radial
A
  1. Answer: C

Source: Day MR, Board Review 2006

142
Q
142. Which of the following are thought to be pain insensitive
structures in the lung?
1. bronchi
2. visceral pleura
3. parietal pleura
4. Lung parenchyma
A
  1. Answer: C (2 & 4)
    Explanation:
    (Raj, Practical Management of Pain, 3rd Ed., page 618)
  2. The trachea and bronchial tree send afferent input
    through the vagus and upper thoracic sympathetics (T2-7)
    and is pain radiates to the sternum.
  3. The visceral pleura has no pain sensation.
  4. The parietal pleura transmits pain along somatic nerves,
    such as the brachial plexus (C8,T1), intercostal nerves
    (T1-12), and phrenic nerves (C3-5).
  5. The lung parenchyma is pain insensitive.
    Source: Shah RV, Board Review 2005
143
Q
  1. The anterior surface of the head of each rib is connected
    to the sides of the bodies of two adjacent vertebrae by the
    following ligament.
  2. Intraarticular
  3. Superior costotransverse
  4. Lateral costotransverse
  5. Radiate
A
  1. Answer: D (4 Only)
    Explanation:
  2. The intraarticular ligament consists of a short fl at band
    of fi bers attached at one end to the crest separating the two
    articular facets on the head of the rib and at the other end
    tothe intervertebral disk.
  3. The superior costotransverse ligament is attached to the
    superior border of the neck of the rib and passes laterally
    to the lower border of the neck of the transverse process
    immediately above.
    3.The radiate ligament connects the anterior part of the
    head of each rib with the sides of the bodies of two
    adjacent vertebrae and the intervertebral disks between
    them.
  4. The lateral costotransverse ligament passes from the
    apex of the transverse process of the vertebra to the rough
    and nonarticular portion of the tubercle of the
    corresponding rib.
144
Q
  1. The superior hypogastric plexus is:
  2. a bilateral structure
  3. located at the level of the third lumbar vertebra
  4. in the proximity of the bifurcation of the common iliac
    vessels
  5. a purely sympathetic chain
A
  1. Answer: B (1 & 3)

Source: Nader and Candido – Pain Practice. June 2001

145
Q
  1. The following statements are true of spinal nerves
  2. they exit the spinal canal at the intervertebral foramina
  3. cervical spinal nerves form the intercostal nerves
  4. they are derived from ventral and dorsal roots of the
    spinal cord
  5. the dorsal rami combine to form plexuses at the cervical
    and lumbosacral levels
A
  1. Answer: B (1 & 3 )
    Explanation:
  2. They exit the spinal canal through the intervertebral
    foramina and immediately split into dorsal and ventral
    rami.
    - The dorsal rami pass posteriorly to innervate the
    paraspinal muscles and skin.
  3. Thoracic spinal nerves form intercostal nerves – not
    cervical
  4. Spinal nerves are derived from the union of the
    corresponding ventral and dorsal roots of the spinal
    cord.
  5. The ventral rami combine to form plexuses at cervical
    and lumbosacral levels. The thoracic spinal nerves from
    the intercostal nerves.
146
Q
  1. The intervertebral disc is composed of all of the following
  2. Nucleus pulposus
  3. Annulus fi brosis
  4. Posterior longitudinal ligament
  5. The end plates
A
  1. Answer: A (1, 2, & 3 )

Source: Rozen. Pain Practice: SEP 2001

147
Q
  1. Which of the following are innervated by fi bers traversing celiac plexus?
  2. Pancreas
  3. Kidney
  4. Duodenum
  5. Descending colon
A
  1. Answer: A (1, 2, & 3)

Source: Boswell MV, Board Review 2005

148
Q

148.Which of the following factors induce pain in visceral
structures?
1. Traction or compression of ligaments, vessels, or mesentery
2. Rapid stretching of the capsule of solid visceral organs
3. Ischemia of visceral musculature
4. Crushing or burning

A
  1. Answer: A (1, 2 & 3)
    Explanation:
    Raj and Patt. Chapter 11. Visceral Pain. In: Pain Medicine:
    A Comprehensive Review, 2nd Edition, Raj, Mosby, 2003
    Source: Boswell MV, Board Review 2005
149
Q
  1. Which of the following is (are) true regarding the superior hypogastric plexus?
  2. Adjacent to the bifurcation of the aorta
  3. Located at the lower 1/3 of L5 vertebral body
  4. At the upper 1/3 of S1 vertebral body
  5. Also named the presacral nerve
A
  1. Answer: C (2 & 4)

Source: Boswell MV, Board Review 2005

150
Q
150.Which of the following are indications for a superior
hypogastric plexus blockade?
1. Bladder pain
2. Ovarian pain
3. Vulvar pain
4. Kidney pain
A
  1. Answer: C (2 & 4)
    Explanation:
  2. The bladder is innervated by the pelvic splanchnics
    (pain afferents with parasympathetics from the sacral
    roots).
  3. Superior hypogastric plexus block is indicated for
    ovarian pain.
  4. The vulva is innervated by somatic fi bers that travel
    with pudendal nerves.
  5. Superior hypogastric plexus block is indicated for
    kidney pain.
    Source: Boswell MV, Board Review 2005
151
Q
151.The musculocutaneous nerve innervates which of the
following muscles?
1. Brachialis
2. Brachiradialis
3. Biceps brachii
4. Tricep
A

.151. Answer: B

Source: Day MR, Board Review 2005

152
Q
  1. Which of the following are true?
  2. Average spinal cord diameter at C4-5 is about 9-11 millimeters
  3. With neck extension, the posterior longitudinal ligament
    is stretched
  4. The ligamentum fl avum reinforces the anterior aspect
    the cervical facet joint capsule
  5. The lateral atlanto-axial joints are responsible for nodding
A
  1. Answer: B
    Explanation:
    (Bonica, 3rd Ed., pages 971-976)
    The average spinal cord diameter from C2-6 is about 10
    millimeters and below C6 it is 7-9 millimeters. With neck
    extension, the posterior longitudinal ligament relaxes and
    the anterior longitudinal ligament stretches. The
    ligamentum fl avum looks like shingles on a roof: they span
    the anterior inferior surface of the cephalad lamina to the
    posterior superior margin of the caudad lamina. They are
    elastic and stretch laterally to reinforce the anterior aspect
    of the zygapophyseal joints. The lateral atlanto-axial joints
    contribute to rotation (the ‘NO’ joint) and the atlantooccipital
    joint contributes to fl exion-extension (nodding,
    the ‘YES’ joint).
    Source: Shah RV, Board Review 2005
153
Q
153.Which of the following cervical vertebra have three
articulating surfaces?
1. C6
2. C2
3. C3
4. C1
A
  1. Answer: D
    Explanation:
    (Bonica, 3rd Ed., page 969)
    The atlas or C1 vertebra does not have a body. It is a solid
    ring of bone with two lateral pillars; the upper and lower
    surfaces articulate with the occiput and C2 vertebra
    respectively. The short anterior arch of C1 articulates with
    the odontoid process of C2 in the vertical plane. The C2
    vertebra has four articulating surfaces. Excluding those
    two already mentioned, the inferior facet articulates with
    the superior facet of C3 and the posterior aspect of the
    dens articulates with the transverse alar ligament. The
    C3 and C7 vertebral bodies have only two each. The
    intervertebral disc and the uncinate processes are not true
    articular surfaces.
    Source: Shah RV, Board Review 2005
154
Q
  1. If you use the external occipital protuberance as a point
    of reference and march anteriorly, you will encounter
    several nerves innervating the cranium. Which of the
    following sequences would be correct?
  2. Least occipital nerve, Greater occipital nerve, Lesser occipital
    nerve, Greater auricular nerve
  3. Lesser occipital nerve, Greater occipital nerve, Least occipital
    nerve, Auriculotemporal nerve
  4. Least occipital nerve, Lesser occipital nerve, Greater
    auricular nerve, Auriculotemporal nerve
  5. Greater occipital nerve, Least Occipital Nerve, Auricolotemporal,
    Greater auricular nerve
A

.154. Answer: B (1 & 3)
Explanation:
(Raj, Pain Review 2nd Ed., page 229-231, Netter’s
Anatomy Atlas)
The correct order is least occipital nerve (C3), greater
occipital nerve (C2), lesser occipital (C2-3), greater
auricular nerve (C2-3), auriculotemporal (V3),
supraorbital (V1), and supratrochlear(V1).
Source: Shah RV, Board Review 2005

155
Q
  1. True statements regarding the pudendal nerve include
  2. it is derived from the S2,S3, and S4 nerves
  3. it leaves the pelvic cavity through the greater sciatic
    foramen
  4. it receives sympathetic fi bers from the sacral portion of
    the sympathetic trunk
  5. it divides into fi ve main branches
A
  1. Answer: A (1, 2, & 3 )
    Explanation:
  2. The somatic fi bers of the pudendal nerve are derived
    from the anterior primary divisions of the S2, S3 and S4
    nerves.
  3. The pudendal nerve leaves the pelvic cavity by passing
    through the greater sciatic foramen inferior to the
    piriformis muscle, between it and the coccygeal muscle.
  4. The sympathetic fi bers of pudendal nerve are
    contributed by the sacral portion of the sympathetic pain.
  5. The pudendal nerve trunk divides into three main
    branches: the inferior hemorrhoidal nerve, the perineal
    nerve, and the dorsal nerve to the clitoris (or to the penis).
156
Q
  1. True Tietze’s syndrome is a condition that includes all of
    the following except:
  2. may be confused with myocardial ischemia
  3. affl icts patients of all ages, but usually patients younger
    than 40
  4. may be associated with bulbous swelling of the costal
    cartilages
  5. involves the lower thoracic costo-transverse joints
A
  1. Answer: D (4 only)
    Explanation:
    (Raj, Practical Management of Pain, 3rd Ed., page 620-
    621)
  2. Although Tietze’s syndrome is synonymous with
    costochondritis (infl ammation of the anterior joints
    corresponding to articulation between the
    sternum/manubrium and ribs), it is specifi c for a
    unilateral anterior chest pain involving the 2nd and 3rd
    anterior costal cartilages. A better term would be
    costochondral pain. This pain develops following blunt
    chest trauma, coughing due to upper respiratory
    infections,and overuse (washing windows or painting). It
    can be confused with myocardial infarction.
  3. It affl icts patients of all ages, but usually those less than
    40 years of age.
  4. Bulbous swellings may persist for several months and
    point tenderness over the costochondral joints are
    common. Treatment is conservative: NSAIDS, TENS, local
    infi ltration, hot/cold packs, electroacupuncture.
    True costochondritis refers to infl ammation and
    arthritides of the costochondral joints at multiple
    locations. This affl icts elderly patients, but the treatment is
    similar to Tietze’s syndrome.
    Source: Shah RV, Board Review 2005
157
Q
  1. Characteristics of the lumber vertebrae include
  2. the body is wider
  3. they are the largest of the immovable vertebrae
  4. the lumbar vertebral (spinal) canal is larger than cervical
    canal
  5. the lumbar vertebral (spinal) canal is larger than the thoracic canal
A
  1. Answer: C (2 & 4)
    Explanation:
  2. The laminae of lumbar vertebrae are broad, short, and
    strong.
    - The body of the vertebra is large and is wider
    transversely than anteroposteriorly and a little thicker anteriorly than posteriorly.
  3. The lumbar vertebrae are the largest of the true
    immovable vertebrae and are also large in comparison to
    their own vertebral canal.
  4. The lumbar vertebral (spinal) canal is triangular and is
    larger than the thoracic canal but smaller than the cervical
    canal.
  5. The lumbar vertebral (spinal) canal is triangular and is
    larger than the thoracic canal but smaller than the cervical
    canal.
158
Q
  1. Which of the following is true about nervus intermedius:
  2. neuralgia of this structure is also known as the otalgic
    type of geniculate neuralgia
  3. it causes pain originating in the ear but radiating to deep
    facial structures
  4. it can be lesioned alone or in conjunction with the following
    cranial nerves: VI, XI, XII.
  5. surgical exposure to section this nerve is easier than that
    for trigeminal or glossopharyngeal structures
A
  1. Answer: A (1,2, & 3)
    Explanation:
    (Raj. Pain Review 2nd Ed., page 313, Bonica 3rd Ed., page
    938)
    Ramsay-Hunt syndrome is the reactivation of herpes
    zoster in the geniculate ganglion and can lead to ear and
    facial pain, rash over the face, hearing loss, and balance
    diffi culties.
  2. The otalgic variety of geniculate neuralgia is known.
  3. Pain can occur in the ear and radiate to the face.
  4. It can be lesioned alone or in conjunction with the
    following cranial nerves: VI, XI, XII.
  5. Surgical exposure of the nervus intermedius is more
    diffi cult than performing trigeminal or glossopharyngeal
    nerve sectioning. Additional cranial nerves that may be
    sectioned include VII, VIII, IX, and X.
    Source: Shah RV, Board Review 2005
159
Q

159.Which of the following do not share a common
innervation with the temporomandibular joint?
1. masseter
2. lateral pterygoid
3. temporalis
4. buccinator

A
  1. Answer: D (4 only)
    Explanation:
    (Raj, Practical Management of Pain 3rd Ed., page 580)
    1, 2, 3. The trigeminal ganglion is commonly thought of as
    only a sensory neural structure, but in reality it provides
    motor innervation to the muscles of mastication: masseter,
    medial and lateral pterygoids, temporalis, mylohyoid and
    anterior belly of the digastric muscles. The temporomandibular
    joint is innervated by the maxillary nerve.
  2. The facial nerve provides muscle innervation to the
    muscles of the face and taste sensation vis a vis chorda
    tympani. Recall the pneumonic:’To Zanzibar By Motor
    Car’, which describes the branches of the facial nerve. The
    buccinator is innervated by the buccal nerve.
    Temporalis
    Zygomatic
    Buccal
    Marginal Mandibular
    Chorda Tympani
    Source: Shah RV, Board Review 2005
160
Q
  1. True statements regarding the epidural space include that
    it is
  2. bound anteriorly by the posterior longitudinal ligament
  3. triangular in the cervical region
  4. bound posteriorly by the ligamentum fl avum
  5. most narrow posteriorly
A
  1. Answer: B (1 & 3)
    Explanation:
  2. The epidural space is bound anteriorly by the posterior
    longitudinal ligament and the vertebral bodies, and laterally by the pedicles and intervertebral foramina.
  3. In the lumbar region, the epidural space is triangular
    with the apex of the triangle corresponding to the
    posterior midline of the vertebral canal.
  4. Posterior boundaries of the epidural space include the
    laminae and ligamenta fl ava, while its inferior boundary is
    its continuation with the sacral canal.
  5. The size of the epidural space varies greatly. The
    anterior portion is the narrowest (approximately 1 mm).
161
Q
  1. Landmarks used in performing a deep cervical plexus
    block include the
  2. mastoid process
  3. cricoid cartilage
  4. Chassaignac’s tubercle (C6)
  5. posterior border of sternocleidomastoid muscle
A
  1. Answer: A (1, 2, & 3 )
    Explanation:
    1, 2 & 3. Insertion sites are located by reference to a line
    that joins the tip of the mastoid process with Chassaignac’s
    tubercle of C6, which is palpated at the level of the cricoid
    cartilage.
    - The deep cervical plexus is composed of the C2-C4
    spinal nerves as they emerge from the foramina in the
    cervical vertebrae.
  2. The posterior border of the sternocleidomastoid muscle
    is the major point of reference used in performing
    blockade of the superfi cial cervical plexus.
162
Q
162.Which of the following brain structures contain high
concentrations of adrenergic neurons?
1. Dorsal raphe
2. Locus ceruleus
3. Striatum
4. Pons
A
  1. Answer: C (2 & 4)
163
Q
  1. The inferior hypogastric Plexus:
  2. Is a unilateral structure
  3. Is situated on either side of the rectum
  4. Provides innervation to the perineum
  5. Provides afferent information from the prostrate
A
  1. Answer: C (2 & 4)

Source: Nader and Candido – Pain Practice. June 2001

164
Q
164. The muscles of anterior abdominal wall include all the
following :
1. cremaster
2. internal oblique
3. pyramidalis
4. external oblique
A
  1. Answer: E (All)
    Explanation:
    The muscles of the abdomen are divided into an
    anterolateral group and a posterior group.
    The anterolateral group is composed of four fl at muscular
    sheets that form the anterior abdominal wall. These
    muscles include the internal and external obliques; the
    transversus and rectus abdominis; and the cremaster and
    pyramidalis muscles, which are involved in suspending the
    testes and tensing the midline tendinous raphe of the
    abdominal wall, respectively.
165
Q

165.Which of the statements regarding spinal structures is
correct:
1. The anterior column consists of vertebral bodies and
intervertebral discs.
2. The pars interarticularis connects the posterior spinal
elements to the vertebral body
3. The posterior spinal elements include the spinous processes, zygapophysial or facet joints, and the lamina.
4. The dens is part of the atlas

A
  1. Answer: B (1 & 3)
    Explanation:
    Reference:
    Gray’s Anatomy, Thirteenth American Edition. Page 127-
    130.
  2. The anterior spinal column consists of vertebral bodies
    and intervertebral discs.
  3. The spinal canal lies between the anterior column and
    the posterior elements and transmits the spinal cord and
    below L2, the cauda equina.
    - The pedicle connects the lamina to the vertebral body,
    thus connecting the posterior and anterior spinal
    structures.
    - The pars interarticularis is part of the lamina between the superior and inferior articular processes.
  4. The posterior elements consist of spinous processes,
    lamina, zygapophysial (facet) joints and posterior spinal
    ligaments (supraspinous, infraspinous and ligamentum
    fl avum).
  5. The dens (odontoid process) is the tooth-like
    projection of the C2 vertebral body (the axis) that juts
    upward into the anterior arch of C1 (the atlas).
    Source: Schultz D, Board Review 2004
166
Q
166. Which of the following nerve entrapments is a potential
cause of eye pain?
1. Supraorbital
2. Occipital
3. Infraorbital
4. Mandibular
A
  1. Answer: A (1, 2, & 3 )
    Explanation:
  2. The supraorbital nerve innervates the eye.
  3. The occipital nerve transmits the referred pain.
  4. Infraorbital nerve innervates the eye.
  5. The mandibular nerve innervates the lower jaw.
    Source: Trescot AM, Board Review 2003
167
Q
  1. True statements with regard to the celiac plexus include
  2. it lies anterior to the crura of the diaphragm
  3. the entire plexus lies anterior to the stomach and omental
    bursa
  4. it is composed of parasympathetic and sympathetic
    fi bers
  5. it lies posterior to the vertebral body of L1
A
  1. Answer: A ( 1, 2, & 3)
    Explanation:
  2. The plexus is situated in the epigastrium just anterior to
    the crura of the diaphragm.
  3. The entire plexus lies posterior to the stomach and the
    omental bursa not anterior.
  4. The celiac plexus is composed of two or more large
    aggregates of ganglion cells, the right and left celiac
    ganglia, a number of smaller ganglia, and a dense network
    of parasympathetic and sympathetic efferent and afferent
    fi bers that enmesh these ganglia.
  5. The plexus is situated anterior to the body of fi rst
    lumbar vertebra not posterior.
168
Q

168.The following are true statements about the pars
interarticularis:
1. It is a part of the pedicle which attaches to the vertebral
body.
2. Spondylolisthesis is the term used to describe a bilateral
pars defect
3. A pars defect is present in approximately 1% of asymptomatic
individuals
4. It is represented by the “neck” of the Scotty dog on
oblique fluoroscopic imaging.

A
  1. Answer: D (4 Only)
    Explanation:
    Bogduk, Clinical Anatomy of the Lumbar Spine and
    Sacrum, Third Edition
  2. The pars interarticularis is a part of the lamina which
    connects the superior and inferior articular processes.
  3. Spondylolysis is the term used to describe a pars defect.
    - Spondylolysis was originally thought to be a defect
    caused by failure of union between two ossifi cation centers
    in the vertebral lamina.
    - Recent evidence however clearly shows that
    spondylolysis is an acquired defect caused by a fatigue
    fracture of the pars interarticularis.
  4. Pars defects are not necessarily painful and are present
    in approximately 10% of asymptomatic individuals.
    - Nonetheless, in the presence of a pars defect, the
    posterior spinal elements are disconnected from the
    anterior spinal column and constitute a “fl ail segment”.
    - This may cause motion instability and the forward movement of one vertebral body on another known as
    spondylolisthesis.
    - The pars defect itself is innervated with free nociceptive
    nerve endings and certainly has the potential to be painful.
    - It has been suggested that infi ltration of the pars defect
    may help to determine whether or not it is involved in
    pain generation.
    - However, no studies have established how often a pars
    defect is the cause for back pain.
  5. Fluoroscopically, the pars interarticularis is represented
    by the neck of the “Scotty dog” and a pars defect appears
    as a “collar” on the dog’s neck.
    Source: Schultz D, Board Review 2004
169
Q
  1. The innervaton of the diaphragm includes
  2. Vagus nerves
  3. Intercostal nerves
  4. Lumbar plexus
  5. Cervical plexus
A
  1. Answer: C (2 & 4)
    Explanation:
  2. Vagus nerves do not innervate the diaphragm.
  3. The diaphragm’s peripheral muscular fi bers are
    supplied by the 6th to the 11th or 12th intercostal nerves.
  4. Lumbar plexus does not innervate the diaphragm.
    4.The most central portion of the diaphragm is innervated
    by the phrenic nerves, which arise from the two cervical
    plexuses (C3-C5).
170
Q
170. Afferent fi bers from the heart enter the central nervous
system via the:
1. vagus nerve
2. greater splanchnic nerve
3. middle cervical ganglion
4. superior cervical ganglion
A
  1. Answer: B (1 & 3)
    Explanation:
    (Raj, Practical Management of Pain 3rd Ed., page 618)
    The visceral afferent fi bers of the heart are transmitted
    through the vagus, cervical ganglia (middle and inferior
    cervical nerves), and the upper fi ve thoracic ganglia
    (thoracic cardiac nerves)…all of which send input to the
    central nervous system via T1-5.
    Source: Shah RV, Board Review 2005
171
Q
  1. Choose the correct statement(s) with respect to sensory
    innervation of the posterior primary division of a spinal
    nerve:
  2. periosteum
  3. Cutaneous and muscular structures
  4. Facet or zygapophysial joints
  5. Posterior longitudinal ligament
A
  1. Answer: A (1, 2, & 3 )
    Explanation:
  2. The posterior primary division provides sensory fi bers
    to periosteum.
  3. The posterior primary division provides sensory fi bers
    to cutaneous and muscular structures.
  4. The posterior primary division provides sensory fi bers
    to facet joints.
  5. The sinuvertebral nerves supply posterior longitudinal
    ligament and other structures within the spinal canal.
172
Q
172. Which of the following are branches of the ophthalmic
division of the trigeminal ganglion?
1. Supratrochlear nerve
2. Infraorbital nerve
3. Frontal nerve
4. Auriculotemporal nerve
A
  1. Answer: B (1 & 3)
    Explanation:
  2. Supratrochlear nerve is a branch of ophthalmic division
    of trigeminal nerve.
  3. Infratrochlear nerve is a branch of maxillary nerve (DR
    M CHECK)
  4. Frontal nerve is a branch of ophthalmic division of
    trigeminal nerve.
  5. Auriculotemporal nerve is a branch of mandibular
    nerve.
    Source: Shah RV, Board Review 2003
173
Q
  1. Ilioinguinal nerve entrapment:
  2. courses in an L1-L2 nerve distribution
  3. Has an S2-S4 nerve distribution
  4. May be noticed postoperatively
  5. Is usually of a cyclical nature
A
  1. Answer: B (1 & 3)

Source: Nader and Candido – Pain Practice. June 2001

174
Q
  1. Scenario:A needle is placed into the spinal canal of a
    cadaver. The needle enters the skin in the mid-sagittal
    plane of the spine and travels through the spinal canal
    from the midline posterior to the midline anterior.The
    following statements are true:
  2. With regards to ligaments, the needle would cross the
    interspinous ligament fi rst, the supraspinous ligament
    second and the ligamentum fl avum third.
  3. The needle would encounter the posterior longitudinal
    ligament only after penetrating the entire thickness of
    spinal cord.
  4. The needle tip would encounter cerebrospinal fl uid
    immediately after penetrating the dura mater and just
    prior to penetrating the arachnoid membrane.
  5. The needle would enter the dorsal epidural space immediately
    after exiting the ligamentum fl avum
A
  1. Answer: C (2 & 4)
    Explanation:
    Reference:
    Gray’s Anatomy, Thirteenth American Edition. Pages 345-
    350.
  2. The posterior spinal ligaments consist of the
    supraspinous, the interspinous and the ligamentum
    fl avum. The supraspinous is the most superfi cial and
    connects the apices of the spinous processes from the
    C7 to the sacrum. The interspinous ligament connects
    adjacent spinous processes with one another and is
    interposed between the supraspinous and the ligamentum
    fl avum. The ligamentum fl avum is markedly elastic and
    connects the lamina of one segment to the adjacent
    lamina. The ligamentum fl avum is the roof of the dorsal
    epidural space.
  3. The posterior longitudinal ligament runs the length of
    the spinal column from the axis to the sacrum. This
    ligament is within the spinal canal and is just anterior to
    the spinal cord and anterior epidural space. The posterior
    longitudinal ligament separates the intervertebral disc
    annulus from the spinal canal.
    - The anterior longitudinal ligament lies on the ventral
    surface of the vertebral bodies extending from the axis to
    the sacrum.
  4. The layers of the spinal canal consist of the dura mater,
    the arachnoid and the pia mater. The dura is a substantial
    fi brous tissue that is the inner boarder of the epidural
    space.
    - Once the dura is penetrated the needle tip enters a
    potential space called the subdural space. This space is
    fi lled with a small amount of serosanguinous fl uid. The
    subdural space is often transcended entirely with quick
    penetration of the thin and closely apposed arachnoid
    membrane which allows access to the cerebrospinal fl uid
    compartment. The pia mater is closely adherent to the
    spinal cord and the CSF fl ows between the pia and the
    arachnoid.
  5. The epidural space is a circular space which surrounds
    the spinal canal and its contents.Posteriorly in the midline,
    the dura is the fl oor of the epidural space and the
    ligamentum fl avum is the roof.
    Source: Schultz D, Board Review 2004
175
Q
  1. The following nerve(s) are branch or branches of the
    maxillary nerve contained within the pterygopalatine
    fossa:
  2. Greater Palatine
  3. Nasopalatine
  4. Superior Alveolar
  5. Inferior Alveolar
A
  1. Answer: A (1, 2, & 3 )
    Explanation:
    1, 2 & 3. Branches of the maxillary nerve within the
    pterygopalatine fossa include the pharyngeal branch to the
    mucosa of the pharynx, the greater palatine branch to the mucosa of the posterior palate, the nasopalatine branch to
    the septal mucosa through the incisive canal to the
    anterior hard palate, and the superior alveolar branch to
    the second and third maxillary molars.
  2. The inferior alveolar nerve is a sensory branch of the
    mandibular nerve that supplies the mandibular teeth,body
    of the mandible, and labial gingiva anterior to the bicuspid
    teeth
176
Q
  1. The visceral pleura receives innervation from
  2. intercostal nerves
  3. sympathetic fi bers with vasomotor function
  4. primary nociceptive afferent fi bers
  5. parasympathetic fi bers via the pulmonary plexus
A
  1. Answer: C (2 & 4)
    Explanation:
  2. The parietal pleura is supplied by the intercostal nerves
    at its lateral aspects, by the T1 spinal nerve at its apex, and
    by the phrenic nerves on the diaphragmatic surface.
  3. The visceral pleura is supplied by sympathetic fi bers that
    have a vasomotor function.
  4. It has afferent fi bers that do not have a nociceptive
    function, making it insensitive to noxious stimuli.
  5. Visceral pleura receives parasympathetic fi bers through
    the pulmonary plexuses.
177
Q
177. A mandibular nerve block at the coronoid notch may
result in the following:
1. Anesthesia of the cornea
2. Loss of sensation to the tongue
3. A subarachnoid injection
4. Anesthesia to the chin
A
  1. Answer: C (2 & 4)
    Explanation:
  2. A trigeminal nerve block at the foramen ovale can result
    in anesthesia to the eye.
  3. The mandibular nerve block at the coronoid will
    include the lingular nerve which gives sensation to the
    tongue resulting in loss of sensation.
  4. A trigeminal nerve block at foramen ovale can result in
    subarachnoid injection.
  5. The mandibular nerve block at coronoid notch will
    block the mental nerve which innervates the chin.
    Source: Trescot AM, Board Review 2003
178
Q
  1. Choose all items that correctly match anatomic structures with their level of termination in adults.
  2. Spinal cord, L1-L2
  3. Preganglionic sympathetic nerves, L2
  4. Spinal canal, sacral hiatus
  5. Dural sac, S4
A
  1. Answer: A (1, 2, & 3)
    Explanation:
  2. In adults the spinal cord ends at L1-L2.
  3. The sympathetic nerve fi ber originates in the
    intermediolateral grey column of the T1-L2 spinal
    segments.
  4. The spinal canal originates at the foramen magnum and
    terminates at the sacral hiatus.
  5. The dural sac terminates at S2 in adults – not S4.
179
Q

179.When performing an interlaminar epidural injection,
each of the following structures is traversed
1. supraspinous ligament
2. ligamentum fl avum
3. interspinous ligament
4. posterior longitudinal ligament

A
  1. Answer: A (1, 2, & 3 )
    Explanation:
    The supraspinous ligament, interspinous ligament, and
    ligamentum fl avum are traversed, after which continued
    advancement places the needle in the epidural space.
  2. The supraspinous ligament is a strong, fi brous cord that
    connects the spinous processes from the C7 vertebra to
    the sacrum. The needle passes through this ligament
    during an interlaminar epidural injection.
  3. The ligamentum fl avum connects the laminae of
    adjacent vertebrae. The needle passes through this
    ligament during an interlaminar epidural injection.
  4. The interspinous ligaments are thin and membranous
    and connect adjoining spinous processes, extending from
    the root to the apex of each process. The needle passes
    through this ligament during an interlaminar epidural
    injection.
  5. The posterior longitudinal ligament extends from the
    axis to the sacrum and passes over the dorsal surface of the
    bodies of the vertebrae and the intervertebral disks.
    - Although the posterior longitudinal ligament is the
    anterior boundary for the epidural space,it is not traversed
    when an epidural injection is performed.
    - It will only be traversed if the needle went through the
    spinal cord.
180
Q
180. All the following nerves pass in front of the ear to provide
innervation of the scalp
1. supraorbital nerve
2. auriculotemporal nerve
3. supratrochlear nerve
4. zygomaticotemporal nerve
A
  1. Answer: E (All)
    Explanation:
    - There are eight nerves involved in processing sensory
    information to the scalp.
    - Four sensory nerves pass in front of the ear to the scalp:
    the supratrochlear and supraorbital nerve from V1
    (ophthalmic branch of trigeminal); the
    zygomaticotemporal nerve from V2 (maxillary branch of
    trigeminal); and the auriculotemporal nerve from V3
    (mandibular branch of trigeminal).
    -Four nerves pass behind the ear: the great auricular nerve
    and the greater, lesser, and least occipital nerves from the
    cervical plexus.
    - All eight nerves converge toward the vertex of the
    scalp.
  2. Supraorbital nerve from ophthalmic branch of
    trigeminal also passes in front of ear.
  3. Auriculotemporal nerve, a branch of mandibular branch
    of trigeminal nerve also passes in front of the ear.
  4. Supratrochlear nerve from ophthalmic branch of
    trigeminal passes in front of the ear.
  5. Zygomatico temporal nerve from maxillary branch of
    trigeminal also passes in front of the ear.
181
Q
  1. The following statements regarding the sacral canal are
    true
  2. it communicates laterally with the sacral foramina
  3. the volume of sacral canal exceeds 100 mL
  4. it contains the cauda equina
  5. its superior border is the sacral hiatus
A
  1. Answer: B (1 & 3)
    Explanation:
  2. The sacral canal is the continuation of the lumbar spinal
    canal. It communicates laterally with the anterior and
    posterior sacral foramina.
  3. The volume of the sacral canal, including the sacral
    foraminal extensions, varies between 12 and 65 mL with a
    mean of about 30 to 34 mL.
  4. The canal contains the fi ve sacral nerve roots and the
    coccygeal nerve, which constitute the cauda equina. Sacral
    canal also contains the fi lum terminale, which exits
    through the sacral hiatus to attach to the back of the
    coccyx.
  5. Inferiorly, the sacral canal terminates at the sacral
182
Q
  1. Abdominal autonomic plexuses include all the following:
  2. celiac plexus
  3. inferior hypogastric plexus
  4. superior hypogastric plexus
  5. esophageal plexus
A
  1. Answer: A (1, 2, & 3 )
    Explanation:
    The abdomen contains three large plexuses composed of
    prevertebral sympathetic ganglia, parasympathetic fi bers
    from the vagus or sacral parasympathetics, and visceral
    afferent fi bers.
    1 The celiac plexus innervates the abdominal viscera.
  2. The inferior hypogastric plexus also supplies the pelvic
    viscera.
  3. The superior hypogastric plexuses supply the pelvic
    viscera.
  4. Esophageal plexus is not (T-T5) abdominal autonomic
    plexus
183
Q
  1. Which of the following statements are ture regarding the brainstem?
  2. It is made up of the medulla, pons and midbrain
  3. It is roofed by the fourth ventricle
  4. It is bounded rostrally by the thalamus
  5. It is bounded ventrally by the clivus
A
  1. Answer: E (All)
    Explanation:
  2. The brainstem is the most caudal portion of the brain
    and consists of the medulla (myelencephalon), pons
    (metencephalon), and midbrain (mesencephalon).
  3. The brainstem is roofed by the fourth ventricle.
  4. The brain stem is bounded rostrally by the thalamus.
  5. The brain stem is bounded ventrally by a fl at bone called
    the clivus.
    - It is bounded dorsally by the fourth ventricle and
    cerebellum caudally by the spinal cord.
184
Q
184. Blockade of the sympathetic efferent activity can be
accomplished by which of the following?
1. Intravenous regional anesthesia
2. Intraspinal local anesthetic
3. Perivascular infi ltration
4. Somatic nerve block
A
  1. Answer: E (All)
    Explanation:
    Sympathetic fi bers can be blocked at all locations. Raj,
    Practical Management of Pain, 3rd edition, page 655
185
Q
  1. Correct statements regarding the sympathetic nervous
    system include the following.
  2. It has cell bodies of preganglionic neurons in the anterolateral
    quadrant of the spinal cord
  3. It has lumbar ganglia that lie on the posterolateral surface
    of the respective vertebrae
  4. It has axons that pass via posterior spinal roots to reach
    paravertebral ganglia
  5. It has paravertebral ganglia that extend from the second
    cervical vertebra to the coccyx
A
  1. Answer: D (4 Only)
    Explanation:
  2. Cell bodies of preganglionic sympathetic neurons lie
    within the intermediolateral gray of spinal cord segments
    (T1-L2).
  3. Lumbar ganglia lie on the anterolateral surface of the
    respective vertebrae.
  4. Axons from these preganglionic neurons pass by way of
    anterior spinal roots and rami communicantes to reach
    paravertebral ganglia of the sympathetic chain.
  5. Paravertebral ganglia are segmentally arrayed in
    bilateral vertical rows extending from the second cervical
    vertebra to the coccyx.
186
Q

186.The true statements with regard to the superior
hypogastric plexus include the following
1. is located at the level of the aortic bifurcation
2. often lies to the right of the midline
3. contains parasympathetic fi bers derived from the pelvic
splanchnic nerves
4. receives contributions from the L5 and S1 splanchnic
nerves

A
  1. Answer: B (1 & 3 )
    Explanation:
  2. The superior hypogastric plexus is situated in front of
    the bifurcation of the abdominal aorta,the body of the
    fi fth lumbar vertebra, and the promontory of the sacrum.
  3. Superior hypogastric plexus often lies in the
    extraperitoneal connective tissue, often to the left of the
    midline - not to the right.
  4. In addition to the sympathetic fi bers that descend to
    form the superior hypogastric plexus, the superior
    hypogastric plexus contains parasympathetic fi bers
    derived from the pelvic splanchnic nerves, which ascend
    from the inferior hypogastric plexus.
  5. The superior hypogastric plexus is formed by the union
    of branches from the aortic plexus with contributions by
    the L3 and L4 splanchnic nerves.
187
Q
187. Landmarks used in performing a superior laryngeal nerve
block include the
1. Transverse process of C6
2. Cricoid cartilage
3. Angle of the mandible
4. Greater cornu of the hyoid cartilage
A
  1. Answer: C (2 & 4)
188
Q
  1. Serotonergic neurons are found predominately in the
  2. Dorsal raphe
  3. Locus ceruleus
  4. Limbic system
  5. Substantia nigra
A
  1. Answer: B (1 & 3)
189
Q
  1. Landmarks for the caudal block include the
  2. Sciatic notch
  3. Posterior-superior iliac spines
  4. Iliac crests
  5. Sacral cornu
A
  1. Answer: C (2 & 4)
190
Q
  1. What are the requirements for single system physical
    examination?
  2. For musculoskeletal system examination primary system
    is musculoskeletal
  3. For neurological system examination primary system is
    nervous system
  4. Other systems to be examined for musculoskeletal system/
    single system examination include lymphatic and
    psychiatric systems
  5. Other systems for neurological examination include
    psychiatric and cardiovascular systems
A
190. Answer: A (1, 2, & 3)
Explanation:
SINGLE SYSTEM EXAMINATION
MUSCULOSKELETAL
* Primary
- Musculoskeletal
* Other
- Constitutional
- Neurological
- Cardiovascular – Peripheral
- Lymphatic
- Skin
- Psychiatric
NEUROLOGICAL
* Primary
- Neurological
* Other
- Constitutional
- Musculoskeletal
- Cardiovascular
- Eyes
191
Q
  1. The following nerves usually blocked at the ankle do not contain motor fi bers?
  2. Posterior tibial nerve
  3. Saphenous nerve
  4. Deep peroneal nerve
  5. Sural nerve
A
  1. Answer: C (2 & 4)
    Explanation:
    Five nerves are blocked when performing an ankle block.
    The saphenous, superfi cial peroneal, and sural nerves are
    all sensory below the ankle.
  2. The posterior tibial nerve causes fl exion of the toes by
    stimulating the fl exor digitorium brevis muscles and
    abduction of the fi rst toe by stimulating the abductor
    hallucis muscles.
    - The posterior tibial nerve is sensory to most of the
    plantar part of the foot.
  3. Saphenous nerve below ankle does not contain motor
    fi bers.
  4. Stimulation of the deep peroneal nerve causes extension
    of the toes by stimulating the extensor digitorum brevis
    muscles.
    - The deep peroneal nerve has a small sensory branch for
    the fi rst interdigital cleft.
  5. Sural nerve below ankle does not contain motor fi bers.
192
Q
  1. True statements regarding epidural veins include
  2. They communicate with abdominal veins via the intervertebral foramina
  3. They lie in the anterolateral part of the epidural space
  4. They are large and valveless
  5. They are small and valveless
A
  1. Answer: A (1, 2, & 3 )
    Explanation:
  2. By way of the intervertebral foramina at each level, the
    vertebral plexus communicates with thoracic and
    abdominal veins.
    - Marked increases in intraabdominal pressure may
    compress the inferior vena cava while distending the
    epidural veins and increasing fl ow up the vertebrobasilar
    plexus.
  3. The major portion of this plexus lies in the anterolateral
    part of the epidural space, out of reach of a correctly
    placed epidural needle.
  4. Epidural veins are large and valveless. They are part of
    the internal vertebral venous plexus, which drains the
    neural tissue of the spinal cord, CSF, and the bony spinal
    canal.
  5. Epidural veins are large and valveless. They are part of
    the internal vertebral venous plexus, which drains the
    neural tissue of the spinal cord, CSF, and the bony spinal
    canal.
193
Q
  1. Sensory innervation to the larynx is derived from
  2. Internal branch of the superior laryngeal nerve
  3. External branch of the superior laryngeal nerve
  4. Recurrent laryngeal nerve
  5. Glossopharyngeal nerve
A
  1. Answer: A (1, 2, & 3)
    Explanation:
  2. The internal branch of the superior laryngeal nerve
    provides sensory innervation to the larynx above the vocal
    cords.
  3. The external branch of the superior laryngeal nerve
    provides sensory innervation to the anterior subglottic
    mucosa.
  4. The recurrent laryngeal nerve provides sensory
    innervation to the larynx below the vocal cords.
  5. Glossopharyngeal nerve does not provide sensory
    innervation to the larynx. It provides sensory innervation
    to tongue and pharynx. (DR M NEED TO CHECK FOR
    ACCURACY)
194
Q
  1. The skin of the penis is supplied by:
  2. the two dorsal nerves of the penis
  3. the pudendal nerve (S2-S4)
  4. the ilioinguinal nerve (base of the penis)
  5. the iliohypogastric nerve (L 1-L2)
A
  1. Answer: A (1, 2, & 3 )

Source: Nader and Candido – Pain Practice. June 2001

195
Q
  1. Branches of the sciatic nerve include
  2. Posterior tibial
  3. Common peroneal
  4. Sural
  5. Saphenous
A
  1. Answer: A (1, 2, & 3)
    Explanation:
    There are four main nerves in the lower extremity: the
    sciatic, femoral, obturator, and lateral femoral cutaneous.
    1, 2. The sciatic nerve is the largest of the four and divides
    into the posterior tibial nerve and common peroneal nerve
    at the popliteal fossa.
  2. The common peroneal nerve divides further into the
    deep and superfi cial peroneal nerves. The tibial nerve
    divides into the posterior tibial and sural nerves.
    - Thus, four of the fi ve nerves that provide sensory
    innervation to the ankle arise from the sciatic nerve: deep
    peroneal, superfi cial peroneal, posterior tibial and sural.
  3. The saphenous nerve is a branch of the femoral nerve
196
Q
  1. The true statements about cricothyroid muscle include:
  2. Is an extrinsic muscle of the larynx
  3. Receives innervation from the recurrent laryngeal
    nerve
  4. Receives innervation from the internal branch of the
    superior laryngeal nerve
  5. It tenses the vocal cords
A
  1. Answer: D (4 Only)
    Explanation:
  2. The cricothyroid muscle is an intrinsic muscle of the
    larynx.
  3. The cricothyroid is innervated by the external branch of
    the superior laryngeal nerve of the vagus.
  4. The cricothyroid is innervated by the external branch of
    the superior laryngeal nerve of the vagus.
  5. The cricothyroid muscle is the only intrinsic muscle of
    the larynx that tenses the vocal cords.
    - All other intrinsic muscles of the larynx receive motor
    innervation from the recurrent laryngeal nerve.
197
Q
197. Landmarks for the sciatic nerve via a posterior approach
include the
1. Posterior superior iliac spine
2. Coccyx
3. Greater trochanter of the femur
4. Iliac crest
A
  1. Answer: B (1 & 3)
198
Q

198.Recurrent laryngeal nerve paralysis is a recognized
complication of which of the following procedures?
1. Ligation of a patent ductus arteriosus
2. Stellate ganglion block
3. Mediastinoscopy
4. Use of a topical ice slush during hear surgery

A
  1. Answer: A (1, 2, & 3)
199
Q
  1. Which of the following anatomic landmarks are needed in
    order to perform a trigeminal ganglion block?
  2. External auditory meatus
  3. Pupil
  4. Lips
  5. Mandibular notch
A
  1. Answer: A (1, 2, & 3 )
    Explanation:
  2. Needle trajectory should aim superiorly toward the
    anterior aspect of the external auditory meatus (in a lateral
    view).
  3. In the coronal plane the needle should be in the
    directionof the mid-pupillary line.
  4. The needle is inserted 2 – 2.5 cm lateral to lips.
  5. The mandibular notch is used for the infrazygomatic
    sphenopalatine ganglion block
    Source: Shah RV, Board Review 2003
200
Q
  1. Which of the following are true regarding sympathetic
    fi bers that exit the spinal cord and connect with the
    sympathetic chain?
  2. Are preganglionic fi bers
  3. Connect with ventral root fi bers
  4. Travel in white communicating rami
  5. Synapse with up to 30 postsynaptic nerves
A
  1. Answer: E (All)

Source: Raj P, Practical Management of Pain, 3rd Ed.

201
Q
  1. Innervation of the discs include all of the following
  2. Sinuvertebral nerve
  3. Lateral branch
  4. L2 roots
  5. Rami communicantes
A
  1. Answer: D (4 Only)

Source: Rozen. Pain Practice: SEP 2001

202
Q
  1. True statements about the intervertebral disc include the following.
  2. It receives blood supply from vessels in the bodies of
    adjacent vertebrae
  3. It receives nutrients by diffusion after its blood supply
    diminishes
  4. It receives nutrients by alternating compression and
    relaxation of the annulus fi brosus
  5. It becomes avascular after the sixth decade of life
A
  1. Answer: A (1, 2, & 3 )
    Explanation:
  2. Early in life, the blood supply to the disk is from the
    periphery as well as from vessels in the bodies of adjacent
    vertebrae, which grow through the cartilaginous plates and
    run toward but do not reach the nucleus pulposus.
  3. After 3rd decade intervertebral disk receives nutrients
    by diffusion of solutes, lymph and other fl uids through the
    cervical portion, the vertebral end plate, and the annulus
    fibrosus.
  4. Intervertebral disk receives nutrition by alternating
    compression and relaxation of the elastic container.
  5. Shortly after birth, the vascular supply begins to
    diminish and by the third decade of life, the disk is almost
    avascular.
    Source: Kahn and Desio
203
Q
  1. The term fl oating applies to the following ribs.
  2. 1st
  3. 11th
  4. 2nd
  5. 12th
A
  1. Answer: C (2 & 4)
    Explanation:
    - The fi rst seven pairs of the 12 ribs are known as the
    vertebrosternal ribs. They connect dorsally with the
    vertebral column and ventrally with the sternum by means
    of costal cartilages.
    - The remaining fi ve pairs are “false” ribs and consists of
    two types. The 8th to 10th ribs have their cartilages
    attached to the cartilage of the rib above
    (vertebrochondral).
    2 & 4. The 11th and 12th ribs are free at their anterior
    extremities and are referred to as fl oating or vertebral ribs
    because they do not attach to the sternum.
204
Q
  1. Characteristics of the posterior longitudinal ligament
    (PLL) include the following:
  2. It begins to progressively widen below the L1 vertebral
    level
  3. It extends along the posterior surface of the body of the
    vertebra
  4. It is composed of fi bers that are less compact than those
    of the anterior longitudinal ligament
  5. It contributes to the anterior wall of the vertebral canal
A
  1. Answer: C (2 & 4)
    Explanation:
  2. The PLL is broad throughout the length of the vertebral
    column until it reaches the L1 vertebral level, where it
    begins to narrow progressively so that at the L5-S1
    interspace it is one half its original width.
  3. The posterior longitudinal ligament (PLL) extends
    along the posterior surface of the body of the vertebra
    from the cervical axis to the sacrum.
  4. The PLL is composed of longitudinal fi bers that are
    denser and more compact than those of the anterior
    longitudinal ligament.
  5. It contributes to the anterior wall of the vertebral canal.
205
Q
  1. Which of the following joints includes an intra-articular
    disc that creates two joint compartments?
  2. Acromioclavicular joint
  3. Glenohumeral joint
  4. Scapulothoracic joint
  5. Sternoclavicular joint
A
  1. Answer: D (4 Only)

Source: Sizer Et Al - Pain Practice March & June 2003